study guide the cardiovascular system and disorders · study guide the cardiovascular system and...

82

Upload: dotuyen

Post on 20-May-2018

230 views

Category:

Documents


8 download

TRANSCRIPT

Page 1: STUDY GUIDE THE CARDIOVASCULAR SYSTEM AND DISORDERS · study guide the cardiovascular system and disorders ... study guide the cardiovascular system and disorders ... (ekg): pemasangan
Page 2: STUDY GUIDE THE CARDIOVASCULAR SYSTEM AND DISORDERS · study guide the cardiovascular system and disorders ... study guide the cardiovascular system and disorders ... (ekg): pemasangan

Study Guide Cardiovascular System and Disorders

Department of Medical Education - Faculty of Medicine - Universitas Udayana, 2018

2

STUDY GUIDE THE CARDIOVASCULAR SYSTEM AND DISORDERS

Planners

Komang Januartha Putra Pinatih

I Made Junior Rina Artha

Agung Nova Mahendra

I.B. Rangga Wibuti

I Made Putra Swi Antara

Nyoman Wiryawan

Putu Gede Sudira

Contributors

I Made Junior Rina Artha

I Wayan Wita

Agung Nova Mahendra

Ni Putu Ekawati

A.A. Wiradewi

Luh Kamiati

Hendy Wirawan

A.A. Ayu Dwi Adelia Yasmin

I Kadek Susila Surya Darma

I Nyoman Wiryawan

I Made Putra Swi Antara

I Nyoman Semadi

I.B. Rangga Wibuti

Luh Oliva Saraswati Suastika

Eka Guna Wijaya

Lisna Astuti

Pontisomaya Parami

Editors

Putu Gede Sudira I Made Junior Rina Artha

Layout

Rizky Darmawan

First Edition March 2017

Second Edition February 2018

All rights reserved. No part of this publication may be reproduced, stored in a retrieval system,

or transmitted in any form or by any means, electronic, mechanical, photocopying, recording, or

otherwise without prior written permission of the publisher.

Published by Department of Medical Education Medicine Programme, Faculty of Medicine,

Universitas Udayana.

Page 3: STUDY GUIDE THE CARDIOVASCULAR SYSTEM AND DISORDERS · study guide the cardiovascular system and disorders ... study guide the cardiovascular system and disorders ... (ekg): pemasangan

Study Guide Cardiovascular System and Disorders

Department of Medical Education - Faculty of Medicine - Universitas Udayana, 2018

3

CONTENTS

STUDY GUIDE THE CARDIOVASCULAR SYSTEM AND DISORDERS ....................................... 2

CONTENTS........................................................................................................................................... 3

PREFACE ............................................................................................................................................. 5

CURRICULUM MAP............................................................................................................................. 6

GENERAL CURRICULUM OF CARDIOVASCULAR SYSTEM AND DISORDERS ...................... 7

PLANNERS AND LECTURERS ........................................................................................................ 10

FACILITATORS .................................................................................................................................. 11

LEARNING ACTIVITY ........................................................................................................................ 12

IMPORTANT INFORMATIONS ......................................................................................................... 12

STUDENT PROJECT ......................................................................................................................... 13

ARTICLE REVIEW ASSESSMENT FORM ...................................................................................... 15

SELF ASSESSMENT ......................................................................................................................... 16

ASSESSMENT METHOD .................................................................................................................. 16

GENERAL TIME TABLE FOR A AND B CLASSES ........................................................................ 16

TIME TABLE OF CLASSES .............................................................................................................. 17

LEARNING PROGRAMS ................................................................................................................... 23

LECTURE 1......................................................................................................................................... 23

LECTURE 3......................................................................................................................................... 28

LECTURE 4......................................................................................................................................... 31

LECTURE 5......................................................................................................................................... 33

LECTURE 6......................................................................................................................................... 35

LECTURE 7......................................................................................................................................... 38

LECTURE 8......................................................................................................................................... 41

LECTURE 9......................................................................................................................................... 43

LECTURE 10 ...................................................................................................................................... 45

LECTURE 12 ...................................................................................................................................... 52

LECTURE 13a .................................................................................................................................... 57

LECTURE 13b .................................................................................................................................... 59

LECTURE 14 ...................................................................................................................................... 61

LECTURE 15 ...................................................................................................................................... 63

Page 4: STUDY GUIDE THE CARDIOVASCULAR SYSTEM AND DISORDERS · study guide the cardiovascular system and disorders ... study guide the cardiovascular system and disorders ... (ekg): pemasangan

Study Guide Cardiovascular System and Disorders

Department of Medical Education - Faculty of Medicine - Universitas Udayana, 2018

4

LECTURE 16 ...................................................................................................................................... 66

BASIC CLINICAL SKILLS .................................................................................................................. 70

BASIC CLINICAL SKILL 1 ................................................................................................................. 71

BASIC CLINICAL SKILL 2 ................................................................................................................. 73

BASIC CLINICAL SKILL 3 ................................................................................................................. 75

BASIC CLINICAL SKILL 4 ................................................................................................................. 77

EVALUATION FORM OF THE CARDIOVASCULAR SYSTEM AND DISODERS....................... 79

REFERENCES ................................................................................................................................... 82

Page 5: STUDY GUIDE THE CARDIOVASCULAR SYSTEM AND DISORDERS · study guide the cardiovascular system and disorders ... study guide the cardiovascular system and disorders ... (ekg): pemasangan

Study Guide Cardiovascular System and Disorders

Department of Medical Education - Faculty of Medicine - Universitas Udayana, 2018

5

PREFACE

The medical curriculum has become increasingly vertically integrated, with stronger basic

concept and support by clinical examples and cases to help in the understanding of the

relevance of the underlying basic science. Basic science concepts may help in the

understanding of the pathophysiology and treatment of diseases. Cardiovascular system and

disorders block has been written to take account of this trend, and to integrate core aspects of

basic science, pathophysiology, and treatment into a single, easy to use revision aid.

The cardiovascular block will focus on anatomy, histology and physiology of

cardiovascular system, pharmacology of different classes of cardiovascular drugs, symptom and

signs of major cardiovascular disease and its pathophysiology and basic principle concept to

education, prevention, treatment and rehabilitation in cardiovascular system disorder in a

patient, family, and community. This study guide is developed by the academic staffs from

various departments: Pharmacology, Anatomy and Clinical Pathology, Cardiology, Pediatric

Cardiology, Cardiothoracic Surgery, and Medical Rehabilitation, Radiology, and Anesthesia.

The learning process will be carried out for 4 weeks (20 working days) starts from

February 26th, 2018 as shown in the timetable. The final examination will be conducted on April

3rd, 2018 in the form of MCQ. The learning situation includes lecture, individual learning, small

group discussion, plenary session, practice, and clinical skills.

Most of the learning material should be learned independently and discuss in SGD by the

students with the help of a facilitator. The lecture is given to emphasize the most important thing

of the material. In a small group discussion, the students gave learning task to lead their

discussion.

This simple study guide needs more revision in the future so that the planners kindly invite

readers to give any comments and critics for its completion. Thank you.

Planners

Page 6: STUDY GUIDE THE CARDIOVASCULAR SYSTEM AND DISORDERS · study guide the cardiovascular system and disorders ... study guide the cardiovascular system and disorders ... (ekg): pemasangan

Study Guide Cardiovascular System and Disorders

Department of Medical Education - Faculty of Medicine - Universitas Udayana, 2018

6

CURRICULUM MAP

Program or curriculum blocks

10 Senior Clerkship

9 Senior Clerkship

8 Senior Clerkship

7

Health System-

based Practice

(3 weeks)

BCS (1 weeks)

Community-

based practice

(4 weeks)

Evidence-based

Medical Practice

(2 weeks)

Special topics :

Health Ergonomy &

Health

Environment

(2 weeks)

Elective Study

IV (evaluation)

(2 weeks)

Compre

Clinic

Orientation

(Clerkship)

+ medical

ethic

(4 weeks)

18

6

The

Cardiovascular

System and

Disorders

(3 weeks)

BCS (1 weeks)

Medical

Emergency

(3 weeks)

BCS (1 weeks)

The Urinary

System and

Disorders

(3 weeks)

BCS (1 weeks)

The Reproductive

System and

Disorders

(3 weeks)

BCS (1 weeks)

Elective Study

III

(3 weeks)

(KKN) 19

5

Neuroscience

and

neurological

disorders

(3 weeks)

BCS (1 weeks)

The Respiratory

System and

Disorders

(3 weeks)

BCS (1 weeks)

The skin &

hearing system

& disorders

(3 weeks)

BCS (1 weeks)

Special Topic :

- Palliative med

- Complemnt &

Alternative Med.

(2 weeks)

Forensic

Medicine and

Medicolegal

(2 weeks)

Elective

Study II

(2 weeks)

18

4

Musculoskeletal

system &

connective

tissue disorders

(3 weeks)

BCS (1 weeks)

Alimentary

& hepatobiliary

systems &

disorders

(3 Weeks)

BCS (1 weeks)

The Endocrine

System,

Metabolism and

Disorders

(3 weeks)

BCS (1 weeks)

Clinical Nutrition

and Disorders

(2 weeks)

BCS (1 weeks)

The Visual

system &

disorders

(2 weeks)

BCS (1weeks)

18

3

Behavior

Change

and disorders

(3 weeks)

BCS (1 weeks)

Basic Infection

& infectious

diseases

(3 weeks)

BCS (1 weeks)

Immune system &

disorders

(2 weeks)

BCS (1 weeks)

Hematologic

system & disorder

& clinical oncology

(3 weeks)

BCS (1 weeks)

Special Topic

(Andro & aging,

Geriatri, Travel

medicine)

(4 weeks)

19

2

BIOMEDIK III

(4 weeks)

Growth

&

development

(2 weeks)

BCS: (1 weeks)

Medical

communication

(2 weeks)

BCS (1 weeks)

Medical

Professionalism

(2 weeks)

BCS (1 weeks)

Basic

Pharmaceutical

medicine &

drug etics

(2 weeks)

Elective

Study I

(2 weeks)

17

1

Studium

Generale and

Humaniora

(2 weeks)

BIOMEDIK I

(8 weeks)

The cell

as biochemical

machinery

(2 weeks)

BCS (1 weeks)

BIOMEDIK II

(6 weeks)

19

Pendidikan Pancasila & Kewarganegaraan ( 3 weeks )

Page 7: STUDY GUIDE THE CARDIOVASCULAR SYSTEM AND DISORDERS · study guide the cardiovascular system and disorders ... study guide the cardiovascular system and disorders ... (ekg): pemasangan

Study Guide Cardiovascular System and Disorders

Department of Medical Education - Faculty of Medicine - Universitas Udayana, 2018

7

GENERAL CURRICULUM OF CARDIOVASCULAR SYSTEM AND DISORDERS

Aims:

Comprehend the structure, physiology, and pathology of the cardiovascular system.

Interpret the laboratory and imaging examination of the cardiovascular system disorders.

Diagnose and treat the patient with common cardiovascular system disorders.

Plan education, prevention, management and rehabilitation of cardiovascular system

disorders to patient, family, and community.

Learning outcomes:

Concern about the size of problem and diversity of cardiovascular disease in the

community.

Able to describe the structure and function of the cardiovascular system.

Able to interpret the result of examination (physical, laboratory, electrocardiography and

chest imaging).

Able to explore patients with the cardiovascular problem (chest pain/ discomfort,

palpitation, dyspnea and cyanosis).

Able to manage major cardiac diseases (hypertension, heart failure, coronary artery

disease, cor pulmonale, valvular heart disease and congenital heart disease).

Able to manage common vascular and lymphatic diseases (venous insufficiency,

lymphedema, peripheral artery disease).

Able to implement rehabilitation of cardiovascular diseases.

Curriculum contents:

Structure and function of the cardiovascular system.

Physiology of the heart and blood vessels in related with intrinsic conduction system,

cardiac output regulation and regulation of blood flow.

Symptoms and signs of cardiovascular disease.

Pathophysiology of cardiovascular system disorders.

Basic physical, laboratory, electrocardiography and imaging examination.

Interpretation of examination results.

Drugs that commonly used in cardiovascular system disorders (anti-hypertension, anti-

angina, anti-arrhythmia, and heart failure drugs).

Basic principles of education, prevention, treatment and rehabilitation in cardiovascular

system disorders in patient, family, and community.

Curriculum structure:

Structure of curriculum mainly derives from general competences of Indonesian general

practitioner. Those competencies in diagnosing diseases and doing clinical skills should be

mastered by all the general practitioners here. Local values of our institutions are also

considered as added values in this curriculum.

Page 8: STUDY GUIDE THE CARDIOVASCULAR SYSTEM AND DISORDERS · study guide the cardiovascular system and disorders ... study guide the cardiovascular system and disorders ... (ekg): pemasangan

Study Guide Cardiovascular System and Disorders

Department of Medical Education - Faculty of Medicine - Universitas Udayana, 2018

8

NO DAFTAR PENYAKIT SESUAI SKDI 2012 TINGKAT KEMAMPUAN

GANGGUAN DAN KELAINAN PADA JANTUNG

1 Syok (septik, hipovolemik, kardiogenik, neurogenic) 3B

2 Angina pectoris 3B

3 Infark myocard 3B

4 Gagal jantung akut 3B

5 Gagal jantung kronik 3A

6 Cardiorespiratory arrest 3B

7 Takikardi : supraventricular, ventrikular 3B

8 Fibrilasi atrial 3A

9 Fibrilasi ventricular 3B

10 Atrial flutter 3B

11 Ekstrasistol supraventrikuler, ventrikuler 3A

12 Kor pulmonale akut 3B

13 Kor pulmonale kronik 3A

GANGGUAN AORTA DAN ARTERI

14 Hipertensi esensial 4A

15 Hipertensi sekunder 3A

GANGGUAN VENA DAN PEMBULUH LIMFE

16 Tromboflebitis 3A

17 Limfangitis 3A

18 Limfedema (primer, sekunder) 3A

19 Insufiensi vena kronik 3A

Page 9: STUDY GUIDE THE CARDIOVASCULAR SYSTEM AND DISORDERS · study guide the cardiovascular system and disorders ... study guide the cardiovascular system and disorders ... (ekg): pemasangan

Study Guide Cardiovascular System and Disorders

Department of Medical Education - Faculty of Medicine - Universitas Udayana, 2018

9

NO KETERAMPILAN KLINIS SESUAI SKDI 2012 TINGKAT KETERAMPILAN

PEMERIKSAAN FISIK

1 Inspeksi Dada 4

2 Palpasi Denyut Apeks Jantung 4

3 Palpasi Arteri Karotis 4

4 Perkusi Ukuran Jantung 4

5 Auskultasi Jantung 4

6 Pengukuran Tekanan Darah 4

7 Pengukuran Tekanan Vena Jugularis (JVP) 4

8 Palpasi Denyut Arteri Ekstremitas 4

9 Penilaian Denyut Kapiler 4

10 Penilaian Pengisian Ulang Kapiler (Capillary Refill) 4

11 Deteksi Bruits 4

PEMERIKSAAN (FISIK) DIAGNOSTIK

12 Tes (Brodie) Trendelenburg 4

13 Elektrokardiografi (EKG): Pemasangan Dan Interpretasi

Hasil EKG Sederhana (VES, AMI, VT, AF) 4

14 Tes Perthes 3

15 Test Homan (Homan’s Sign) 3

16 Uji Postur Untuk Insufisiensi Arteri 3

17 Tes Hiperemia Reaktif Untuk Insufisiensi Arteri 3

18 Test Ankle-Brachial Index (ABI) 3

RESUSITASI

19 Pijat Jantung Luar 4

20 Resusitasi Cairan 4

Page 10: STUDY GUIDE THE CARDIOVASCULAR SYSTEM AND DISORDERS · study guide the cardiovascular system and disorders ... study guide the cardiovascular system and disorders ... (ekg): pemasangan

Study Guide Cardiovascular System and Disorders

Department of Medical Education - Faculty of Medicine - Universitas Udayana, 2018

10

PLANNERS AND LECTURERS

NO NAME DEPARTMENT PHONE

1. dr. I Made Junior Rina Artha, Sp.JP(K),

FIHA, FAsCC (Coordinator) Cardiology 08123814814

2. dr. Luh Oliva Saraswati Suastika, Sp.JP,

FIHA (Secretary) Cardiology 081330530247

3. Prof. Dr. dr. I Wayan Wita, Sp.JP(K), FIHA,

FAsCC (member) Cardiology 08123809780

4. dr. Putu Gede Sudira, Sp.S (member) DME 081805633997

5. dr. Agung Nova Mahendra, M.Sc Pharmacology 087861030195

6. dr. Putu Ekawati, M.Repro, Sp.PA Pathology Anatomy 08123958158

7. Dr. dr. AA Wiradewi, Sp.PK Clinical Pathology 08155237937

8. dr. I Kadek Susila Surya Darma, M. Biomed,

Sp.JP, FIHA Cardiology 08113853151

9. dr. I.B. Rangga Wibhuti, M.Biomed,

Sp.JP(K), FIHA, FASE Cardiology 081237287888

10. dr. I Made Putra Swi Antara, Sp.JP(K),

FIHA Cardiology 08123804782

11. dr. A.A. Ayu Dwi Adelia Yasmin, M. Biomed,

SpJP, FIHA Cardiology 087861402169

12. dr. I Nyoman Wiryawan, SpJP(K), FIHA Cardiology 081289053234

13. dr. Hendy Wirawan, SpJP Cardiology 0817352649

14. Dr. dr. Semadi, SpB, SpBTKV Cardiothorac Surgery 08123838654

15. dr. Eka Guna Wijaya, Sp.A(K) Pediatric Cardiology 081338599801

16. dr. Lisna Astuti, Sp.Rad. Radiology 081337934497

17. dr. Luh Kamiati, Sp.KFR Physiotherapy 08123998787

18. dr. Pontisomaya Parami, Sp.An., MARS Anesthesia 08113800107

Page 11: STUDY GUIDE THE CARDIOVASCULAR SYSTEM AND DISORDERS · study guide the cardiovascular system and disorders ... study guide the cardiovascular system and disorders ... (ekg): pemasangan

Study Guide Cardiovascular System and Disorders

Department of Medical Education - Faculty of Medicine - Universitas Udayana, 2018

11

FACILITATORS Regular Class (Class A)

No Name Group Departement Phone Venue

(2ndfloor)

1 Dr. dr. Ni Made Linawati, M.Si A1 Histology 081337222567 2nd floor:

R.2.09

2 dr. I Wayan Sugiritama, M.Kes A2 Histology 08164732743 2nd floor:

R.2.10

3 dr. Wayan Citra Wulan Sucipta

Putri, MPH A3 Public Health 087761838141

2nd floor:

R.2.11

4 Dr. dr. Ni Nyoman Sri Budayanti,

Sp.MK (K) A4 Microbiology 08553711398

2nd floor:

R.2.12

5 Prof. Dr. dr. I Made Jawi, M.Kes A5 Pharmacology 08179787972 2nd floor:

R.2.13

6 Dr. dr. Desak Made Wihandani,

M.Kes A6 Biochemistry 081338776244

2nd floor:

R.2.14

7 dr. I Nyoman Gede Wardana,

M.Biomed A7 Anatomy 087860405625

2nd floor:

R.2.15

8 Dr.dr. I Made Muliarta, M.Kes A8 Physiology 081338505350 2nd floor:

R.2.16

9 Dr. dr, I Made Sudarmaja,

M.Kes A9 Parasitology 08123953945

2nd floor:

R.2.20

10 dr. Ni Made Dewi Dian

Sukmawati, Sp.PD A10

Internal

Medicine 08123320380

2nd floor:

R.2.21

English Class (Class B)

No Name Group Departement Phone Venue

(2ndfloor)

1 dr. Putu Gede Sudira Sp.S B1 DME 081805633997 2nd floor:

R.2.09

2 Dr. dr. Bagus Komang

Satriyasa, M.Repro B2 Pharmacology 087777790064

2nd floor:

R.2.10

3 dr. Sari Wulan Dwi Sutanegara,

Sp.T.H.T.K.L (K)., FICS B3 ENT

081237874447/

081338466039 2nd floor:

R.2.11

4 dr. Pontisomaya Parami, Sp. An,

MARS B4 Anesthesia 08113800107

2nd floor:

R.2.12

5 Dr. dr. Susy Purnawati, M.KK B5 Physiology 08123989891 2nd floor:

R.2.13

6 dr. I Wayan Surudarma, Msi B6 Biochemistry 081338486589 2nd floor:

R.2.14

7 dr. Henky, Sp. F, M.Beth,

FACLM B7 Forensics 08123988486

2nd floor:

R.2.15

8 Dr. dr. Cok Bagus Jaya

Lesmana, Sp.KJ (K) B8 Psychiatry 0816295779

2nd floor:

R.2.16

9 dr. I Wayan Gede Sutadarma,

M.Gizi, Sp.GK B9 Biochemistry 082144071268

2nd floor:

R.2.20

10 dr. I.G.A.A. Dwi Karmila, Sp.KK B10 Dermatovenerol

ogy 08123978446

2nd floor:

R.2.21

Page 12: STUDY GUIDE THE CARDIOVASCULAR SYSTEM AND DISORDERS · study guide the cardiovascular system and disorders ... study guide the cardiovascular system and disorders ... (ekg): pemasangan

Study Guide Cardiovascular System and Disorders

Department of Medical Education - Faculty of Medicine - Universitas Udayana, 2018

12

LEARNING ACTIVITY

There are several types of learning activity:

Lecture

Plenary session

Independent learning based on the lecture’s topic

Small group discussion to solve the learning task

Practicing

Student project

Clinical skill and demonstration

Self-assessment at the end of every topic

The lecture will be held at room 4.02 (4th floor), while discussion rooms available at 2nd

floor (room 2.09 - 2.16, 2.20, and 2.21).

IMPORTANT INFORMATION Meeting of the students’ representative

In the middle of the block schedule, a meeting is designed among the student

representatives of every small group discussions, facilitators, and resource persons. The

meeting will discuss the ongoing teaching-learning process, quality of lecturers and facilitators

as a feedback to improve the next process. The meeting will be taken during student project

presentation.

Page 13: STUDY GUIDE THE CARDIOVASCULAR SYSTEM AND DISORDERS · study guide the cardiovascular system and disorders ... study guide the cardiovascular system and disorders ... (ekg): pemasangan

Study Guide Cardiovascular System and Disorders

Department of Medical Education - Faculty of Medicine - Universitas Udayana, 2018

13

STUDENT PROJECT

Title of student project

Group discussion Topic

A1 Endokarditis infeksiosa

A2 Vena varikose

A3 Aneurisma aorta

A4 Kardiomiopati hipertrofi

A5 Koartasio aorta

A6 Trombosis vena dalam

A7 Sindroma Wolff-Parkinson-White (WPW)

A8 Iskemia tungkai akut

A9 Penyakit Raynaud

A10 Hipertensi pulmonal

B1 Perikarditis

B2 Obstructed venous return

B3 Diseksi aorta

B4 Kardiomiopati restriktif

B5 Subclavian steal syndrome

B6 Emboli vena

B7 Sindroma Brugada

B8 Peripheral artery disease (klaudikasio)

B9 Penyakit Buerger

B10 Anomali Ebstein

About Topic, Presentation’s place and schedules, Task rules, Assessment, and Evaluator will

be discussed at a lecture of block introduction on February 26th, 2018.

Page 14: STUDY GUIDE THE CARDIOVASCULAR SYSTEM AND DISORDERS · study guide the cardiovascular system and disorders ... study guide the cardiovascular system and disorders ... (ekg): pemasangan

Study Guide Cardiovascular System and Disorders

Department of Medical Education - Faculty of Medicine - Universitas Udayana, 2018

14

TITLE

(subject/ topic: choose from competency list)

Name

NIM

Faculty of Medicine, Udayana University

2018

______________

1. Introduction (Pendahuluan)

2. Content (Isi, sesuai topik yang dibahas)

3. Summary (Ringkasan)

4. References: (Daftar Pustaka) Van Couver style

Example:

Journal

Sheetz MJ, King GL. Molecular understanding of hyperglycemia’s adverse effect for

diabetic complications. JAMA. 2002;288:2579-86.

Textbook

Libby P. The Pathogenesis of atherosclerosis. In: Braunwald E, Fauci A, Kasper D,

Hoster S, Longo D, Jamason S (eds). Harrison’s principles of internal medicine. 15th ed.

New York: McGraw Hill; 2001. p. 1977-82.

Internet

WHO. Obesity: preventing and managing the global epidemic. Geneva: WHO 1998.

[cited 2005 July]. Available from:

http://www.who.int/dietphysicalactivity/publications/facts/ obesity/en.

Student project consists of 6 – 10 pages, 1.5 space, Times new romance 12.

Page 15: STUDY GUIDE THE CARDIOVASCULAR SYSTEM AND DISORDERS · study guide the cardiovascular system and disorders ... study guide the cardiovascular system and disorders ... (ekg): pemasangan

Study Guide Cardiovascular System and Disorders

Department of Medical Education - Faculty of Medicine - Universitas Udayana, 2018

15

ARTICLE REVIEW ASSESSMENT FORM

Faculty of Medicine, Udayana University

___________________________________________________________________________

Block : Cardiovascular System and Disorders

Name : ________________________________________

Student No. (NIM) : ________________________________________

Facilitator : ________________________________________

Title : ________________________________________

________________________________________

Time table of consultation

Point of discussion Week Date Tutor sign

1. Title 1

2. References 1

3. Outline of paper 2

4. Content 3

5. Final discussion 4

Assessment

A. Paper structure : 7 8 9 10

B. Content : 7 8 9 10

C. Discussion : 7 8 9 10

Total point : ( A + B + C ) : 3 = _____________

Denpasar, ______________________

Facilitator/ Evaluator

Page 16: STUDY GUIDE THE CARDIOVASCULAR SYSTEM AND DISORDERS · study guide the cardiovascular system and disorders ... study guide the cardiovascular system and disorders ... (ekg): pemasangan

Study Guide Cardiovascular System and Disorders

Department of Medical Education - Faculty of Medicine - Universitas Udayana, 2018

16

SELF ASSESSMENT

Self-assessment of each lecture will be given after each lecture session and will be marked.

This mark can determine whether the student passes this block or not. Any final mark between

62 - 64 will be reconsidered with self-assessment’s mark to see the student’s status. Any

student with self-assessment’s mark 65 or more will pass this block. And for the lower one will

have to attend the remedial examination. It is important to do this self-assessment cautiously,

because this activity may be your ticket to pass this block just at first examination.

ASSESSMENT METHOD

Assessment in this theme consists of:

SGD : 5%

Final Exam : 80%

Student Project : 15%

Final mark 65 or more considered to pass this block. Certain conditions applied for those with a

final mark between 62 – 64. These students will be analyzed using their self-assessment’s

mark. Students with final mark 62 – 64 and self-assessment’s mark equal or more than 65 will

also consider passing this block. The value of marking:

A ≥ 80

B+ >70-79

B 65-70

GENERAL TIMETABLE FOR A AND B CLASSES

CLASS A CLASS B

TIME ACTIVITIES TIME ACTIVITIES

08.00-09.00 Lecture 09.00-10.00 Lecture

09.00-10.30 Independent learning 10.00-11.30 Student project

10.30-12.00 SGD 11.30-12.00 Break

12.00-12.30 Break 12.00-13.30 Independent learning

12.30-14.00 Student project 13.30-15.00 SGD

14.00-15.00 Plenary session 15.00-16.00 Plenary session

Page 17: STUDY GUIDE THE CARDIOVASCULAR SYSTEM AND DISORDERS · study guide the cardiovascular system and disorders ... study guide the cardiovascular system and disorders ... (ekg): pemasangan

Study Guide Cardiovascular System and Disorders

Department of Medical Education - Faculty of Medicine - Universitas Udayana, 2018

17

TIMETABLE OF CLASSES

DAY/DATE Class A Class B ACTIVITY VENUE PIC

1

Monday,

February 26

2018

08.00-08.15 09.00-09.15 Introduction Classroom dr. I Made Junior Rina

Artha, SpJP(K)

08.15-09.00 09.15-10.00

Lecture 1 Approach to patient

with cardiovascular

disease

Classroom

Prof. Dr. dr. I Wayan

Wita, SpJP(K)

09.30-10.30 12.00-13.30 Independent learning

10.30-12.00 13.30-15.00 SGD Disc room Facilitator

12.00-12.30 11.30-12.00 Break

12.30-14.00 10.00-11.30 Student Project

14.00-15.00 15.00-16.00 Plenary session Classroom Prof. Dr. dr. I Wayan

Wita, SpJP(K)

2

Tuesday,

February 27

2018

08.00-08.30 09.00-09.30 Lecture 2 Cardiovascular

Pharmacology

Classroom dr. Agung Nova

08.30-09.00 09.30-10.00

Lecture 3 Pathologic Anatomy of

Cardiovascular

System: Anomalies

Formation of the heart

and great vessels

Classroom dr. Ni Putu Ekawati,

SpPA

09.00-10.30 12.00-13.30 Independent learning

10.30-12.00 13.30-15.00 SGD Disc room Facilitator

12.00-12.30 11.30-12.00 Break

12.30-14.00 10.00-11.30 Student Project

14.00-15.00 15.00-16.00 Plenary session Classroom

dr. Agung Nova/dr. Ini

Putu Ekawati, SpPA

Page 18: STUDY GUIDE THE CARDIOVASCULAR SYSTEM AND DISORDERS · study guide the cardiovascular system and disorders ... study guide the cardiovascular system and disorders ... (ekg): pemasangan

Study Guide Cardiovascular System and Disorders

Department of Medical Education - Faculty of Medicine - Universitas Udayana, 2018

18

3

Wednesday,

February 28

2018

08.00-08.30 09.00-09.30

Lecture 4 Laboratory

examination: cardiac

markers

Classroom Dr. dr. AA Wiradewi,

SpPK

08.30-09.00 09.30-10.00 Lecture 5 Cardiac Rehabilitation

Classroom dr. Luh Kamiati,

SpRM

09.00-10.30 12.00-13.30 Independent learning

10.30-12.00 13.30-15.00 SGD Disc room Facilitator

12.00-12.30 11.30-12.00 Break

12.30-14.00 10.00-11.30 Student project

14.00-15.00 15.00-16.00 Plenary session Classroom Dr. dr. AA Wiradewi,

SpPK/ dr. Luh

Kamiati, SpRM

4

Thursday,

March 1

2018

08.00-09.00 09.00-10.00 Lecture 6 Acute coronary

syndrome

Classroom dr. I Made Junior Rina

Artha, SpJP(K)

09.00-10.30 12.00-13.30 Independent learning

10.30-12.00 13.30-15.00 SGD Disc room Facilitator

12.00-12.30 11.30-12.00 Break

12.30-14.00 10.00-11.30 Student project

14.00-15.00 15.00-16.00 Plenary Session Classroom dr. I Made Junior Rina

Artha, SpJP(K)

5

Monday,

March 5

2018

08.00-09.00 09.00-10.00 Lecture 7 Stable coronary artery

disease

Classroom dr. Hendy Wirawan,

SpJP

09.00-10.30 12.00-13.30 Independent learning

10.30-12.00 13.30-15.00 SGD Disc room Facilitator

12.00-12.30 11.30-12.00 Break

12.30-14.00 10.00-11.30 Student project

Page 19: STUDY GUIDE THE CARDIOVASCULAR SYSTEM AND DISORDERS · study guide the cardiovascular system and disorders ... study guide the cardiovascular system and disorders ... (ekg): pemasangan

Study Guide Cardiovascular System and Disorders

Department of Medical Education - Faculty of Medicine - Universitas Udayana, 2018

19

14.00-15.00 15.00-16.00 Plenary session Classroom dr. Hendy Wirawan,

Sp.JP

6

Tuesday,

March 6

2018

08.00-09.00 09.00-10.00 Lecture 8 Hypertension part 1

Classroom dr. AAA Adelia

Yasmin, SpJP

09.00-10.30 12.00-13.30 Independent learning

10.30-12.00 13.30-15.00 SGD Disc room Facilitator

12.00-12.30 11.30-12.00 Break

12.30-14.00 10.00-11.30 Student project

14.00-15.00 15.00-16.00 Plenary session Classroom dr. AAA Adelia

Yasmin, SpJP

7

Wednesday,

March 7

2018

08.00-09.00 09.00-10.00 Lecture 9 Hypertension part 2

Classroom dr. AAA Adelia

Yasmin, SpJP

09.00-10.30 12.00-13.30 Independent learning

10.30-12.00 13.30-15.00 SGD Disc room Facilitator

12.00-12.30 11.30-12.00 Break

12.30-14.00 10.00-11.30 Student project

14.00-15.00 15.00-16.00 Plenary session Classroom dr. AAA Adelia

Yasmin, SpJP

8

Thursday,

March 8

2018

08.00-09.00 09.00-10.00 Lecture 10 Acute Heart Failure

Classroom

dr. Kadek Susila SD,

SpJP

09.00-10.30 12.00-13.30 Independent learning

10.30-12.00 13.30-15.00 SGD Disc room Facilitator

12.00-12.30 11.30-12.00 Break

12.30-14.00 10.00-11.30 Student project

14.00-15.00 15.00-16.00 Plenary session Classroom dr. I Kadek Susila SD,

SpJP

Page 20: STUDY GUIDE THE CARDIOVASCULAR SYSTEM AND DISORDERS · study guide the cardiovascular system and disorders ... study guide the cardiovascular system and disorders ... (ekg): pemasangan

Study Guide Cardiovascular System and Disorders

Department of Medical Education - Faculty of Medicine - Universitas Udayana, 2018

20

9

Friday,

March 9

2018

08.00-09.00 09.00-10.00 Lecture 11 Chronic Heart Failure

Classroom

dr. I Nyoman

Wiryawan, SpJP(K)

09.00-10.30 12.00-13.30 Independent learning

10.30-12.00 13.30-15.00 SGD Disc room Facilitator

12.00-12.30 11.30-12.00 Break

12.30-14.00 10.00-11.30 Student project

14.00-15.00 15.00-16.00 Plenary session Classroom dr. I Nyoman

Wiryawan, SpJP(K)

10

Monday,

March 12

2018

08.00-09.00 09.00-10.00 Lecture 12 Arrhythmia

Classroom dr. Putra Swi Antara,

SpJP(K)

09.00-10.30 12.00-13.30 Independent learning

10.30-12.00 13.30-15.00 SGD Disc room Facilitator

12.00-12.30 11.30-12.00 Break

12.30-14.00 10.00-11.30 Student project

14.00-15.00 15.00-16.00 Plenary Session Classroom dr. Putra Swi Antara,

SpJP(K)

11

Tuesday,

March 13

2018

08.00-09.00 09.00-10.00

Lecture 13 Common Peripheral

Vascular and

Lymphatic disease

Classroom Dr. dr. Semadi,

SpBTKV

09.00-10.30 12.00-13.30 Independent learning

10.30-12.00 13.30-15.00 SGD Disc room Facilitator

12.00-12.30 11.30-12.00 Break

12.30-14.00 10.00-11.30 Student project

14.00-15.00 15.00-16.00 Plenary session Classroom

Dr. dr. Semadi,

SpBTKV

Page 21: STUDY GUIDE THE CARDIOVASCULAR SYSTEM AND DISORDERS · study guide the cardiovascular system and disorders ... study guide the cardiovascular system and disorders ... (ekg): pemasangan

Study Guide Cardiovascular System and Disorders

Department of Medical Education - Faculty of Medicine - Universitas Udayana, 2018

21

12

Wednesday,

March 14

2018

08.00-09.00 09.00-10.00 Lecture 14 Valvular Heart Disease

Classroom dr. IB Rangga W,

SpJP(K)

09.00-10.30 12.00-13.30 Independent learning

10.30-12.00 13.30-15.00 SGD Disc room Facilitator

12.00-12.30 11.30-12.00 Break

12.30-14.00 10.00-11.30 Student project

14.00-15.00 15.00-16.00 Plenary session Classroom dr. IB Rangga W,

SpJP(K)

13

Thursday,

March 15

2018

08.00-09.00 09.00-10.00 Lecture 15 Cor Pulmonale

Classroom

dr. Luh Oliva S

Suastika, SpJP

09.00-10.30 12.00-13.30 Independent learning

10.30-12.00 13.30-15.00 SGD Disc room Facilitator

12.00-12.30 11.30-12.00 Break

12.30-14.00 10.00-11.30 Student project

14.00-15.00 15.00-16.00 Plenary session Classroom dr. Luh Oliva S

Suastika, SpJP

14

Monday,

March 19

2018

08.00-09.00 09.00-10.00

Lecture 16 Congenital heart

disease and Acute

Rheumatic Fever

Classroom dr. Eka Guna Wijaya,

SpA(K)

09.00-10.30 12.00-13.30 Independent learning

10.30-12.00 13.30-15.00 SGD Disc room Facilitator

12.00-12.30 11.30-12.00 Break

12.30-14.00 10.00-11.30 Student project

14.00-15.00 15.00-16.00 Plenary session Classroom dr. Eka Guna Wijaya,

SpA(K)

Page 22: STUDY GUIDE THE CARDIOVASCULAR SYSTEM AND DISORDERS · study guide the cardiovascular system and disorders ... study guide the cardiovascular system and disorders ... (ekg): pemasangan

Study Guide Cardiovascular System and Disorders

Department of Medical Education - Faculty of Medicine - Universitas Udayana, 2018

22

15

Tuesday,

March 20

2018

08.00-15.00 09.00-16.00 Student Project

Presentation

Auditorium/

Classroom* Examiner SP

16

Thursday,

March 22

2018

08.00-11.00 11.00-14.00 Basic Clinical Skill 1: Physical Examination 1

Classroom/

Skill lab

dr. Luh Oliva S

Suastika, SpJP

17

Friday,

March 23

2018

08.00-11.00 11.00-14.00

Basic Clinical Skill 2: ECG

Classroom/

Skill lab

dr. Putra Swi Antara,

SpJP(K)

18

Monday,

March 26

2018

08.00-09.30 09.30-11.00 Basic Clinical Skill 3: Chest radiography

Classroom/

Skill lab

dr. Lisna Astuti,

SpRad

11.00-13.00 13.00-15.00 Basic Clinical Skill 1: Physical Examination 2

Classroom/

Skill lab

dr. IB Rangga W,

SpJP(K)

19

Tuesday,

March 27

2018

08.00-11.00 11.00-14.00

Basic Clinical Skill 4: Cardiopulmonary

resuscitation

Classroom/

Skill lab

dr. I Kadek Susila SD,

SpJP

20

Wednesday,

March 28

2018

08.00-11.00 11.00-14.00 Basic Clinical Skill 5: IV line & venopuncture

Classroom/

Skill lab

dr. Pontisomaya

Parami, Sp.An.,

MARS

22

Friday,

March 30

2018

Pre Evaluation Break

23

Monday,

April 2

2018

Examination

Friday,

July 27

2018

Remedial Examination

Page 23: STUDY GUIDE THE CARDIOVASCULAR SYSTEM AND DISORDERS · study guide the cardiovascular system and disorders ... study guide the cardiovascular system and disorders ... (ekg): pemasangan

Study Guide Cardiovascular System and Disorders

Department of Medical Education - Faculty of Medicine - Universitas Udayana, 2018

23

LEARNING PROGRAMS

LECTURE 1

APPROACH TO PATIENT WITH CARDIOVASCULAR DISEASE

Prof dr. I Wayan Wita, SpJP(K), FIHA, FAsCC

Department of Cardiology and Vascular Medicine

AIMS:

Able to do and practice the approach to the patient with Cardiovascular Disease (common

cardiological symptoms and consultations and investigations in Cardiology).

LEARNING OUTCOME:

1. Able to do and practice the approach to patient common Cardiovascular symptoms.

2. Able to do and practice the approach to consultations with Cardiovascular Disease.

3. Able to do and practice the approach to investigations in Cardiology).

CURRICULUM CONTENT:

1. The symptoms of the cardiovascular disease.

2. The diagnostic tools to confirm patients with Cardiovascular Diseases.

ABSTRACT:

History taking remains the most important component of the diagnostic process. Often diagnosis

can be made from the history alone, with examination and investigations only serving to confirm

it. Chest pain is a common symptom. Breathlessness caused by left ventricle failure may

present as orthopnoea and paroxysmal nocturnal dyspnoea. Palpitation is usually a benign

symptom unless it is accompanied by syncope or presyncope.

The cardiovascular examination begins the moment the patient enters the room. Is the patient

pale, breathless or anxious? Examine the pulse, and check the pulse character. The blood

pressure and auscultation should be performed in an appropriate manner. The

electrocardiogram (ECG) and chest-x-ray remain the most valuable cardiac investigation in

clinical practice. 24-hour ECG recording is most useful in those with very frequent arrhythmia

symptoms. Stress testing is performed for 2 main reasons: to diagnose ischemic heart disease

and to assess prognosis. Echocardiography provides both structural and functional information

that assists in the diagnosis of many cardiac conditions. Cardiac catheterization is an invasive

procedure that assesses systemic and pulmonary hemodynamic variables, as well as oxygen

saturation and intracardiac shunts. It assesses aortic, valvular, left ventricular and coronary

artery structure and function. The assessment of coronary artery disease is the main indication.

Page 24: STUDY GUIDE THE CARDIOVASCULAR SYSTEM AND DISORDERS · study guide the cardiovascular system and disorders ... study guide the cardiovascular system and disorders ... (ekg): pemasangan

Study Guide Cardiovascular System and Disorders

Department of Medical Education - Faculty of Medicine - Universitas Udayana, 2018

24

The imaging investigation of the heart may be considered under the following:

1. Chest X-ray

The chest radiograph was one of the first clinical examinations to use the then-new technology

of diagnostic radiography. It remains the most common x-ray examination and one of the most

difficult examinations to interpret. With careful evaluation, it yields a large amount of anatomic

and physiologic information. Chest X-ray remains the valuable cardiac investigation in clinical

practice.Radiologic method used in the roentgen cardiac examination:

a) Posteroanterior projection, PA/AP

b) Lateral Projection

c) Right anterior oblique projection (RAO)

d) Left anterior oblique projection (LAO).

Increase in cardiac size is the most consistent indication of cardiac disease.

2. Computed tomography (CT-scan)

The basic principle of CT technology is the use of ionizing radiation within a gantry rotating

around the patient in which x-rays are detected on a detector array and converted through

reconstruction algorithms to images. It is these images, acquired at high spatial and temporal

resolution, that have enabled cardiovascular medicine to enter the CT imaging era.

3. Magnetic resonance imaging (MRI)

Over the past decade, cardiac magnetic resonance (CMR) has developed into a routine clinical

imaging tool. With excellent spatial and temporal resolution, unrestricted tomographic fields, and

no exposure to ionizing radiation, CMR offers detailed morphologic and functional

characterization for most types of heart disease.

4. Echocardiography

Echocardiography remains the most frequently used and usually the initial imaging test to

evaluate all cardiovascular diseases related to a structural, functional, or hemodynamic

abnormality of the heart or great vessels. Echocardiography uses ultrasound beams reflected by

cardiovascular structures to produce characteristic lines or shapes caused by normal or altered

cardiac anatomy in one, two, or three dimensions by M (motion)–mode, two-dimensional, or

three-dimensional echocardiography, respectively. Doppler examination and color flow imaging

provide a reliable assessment of cardiac hemodynamics and blood flow.

5. Angiocardiography

Angiography is a technique used to visualize the lumen, of blood vessels and organs of the

body, with a particular interest in the arteries, veins, and the heart chambers. This is traditionally

done by injecting a radio-opaque contrast agent into the blood vessel and imaging using X-ray

based techniques such as fluoroscopy.

6. Cardiac catheterization

Cardiac catheterization is the insertion of a catheter into a chamber or vessel of the heart. This

is done both for diagnostic and interventional purposes. Subsets of this technique are mainly

coronary catheterization, involving the catheterization of the coronary arteries, and

catheterization of cardiac chambers and valves of the Cardiac System.

Page 25: STUDY GUIDE THE CARDIOVASCULAR SYSTEM AND DISORDERS · study guide the cardiovascular system and disorders ... study guide the cardiovascular system and disorders ... (ekg): pemasangan

Study Guide Cardiovascular System and Disorders

Department of Medical Education - Faculty of Medicine - Universitas Udayana, 2018

25

7. Nuclear Cardiology

The era of noninvasive radionuclide cardiac imaging in humans began in the early 1970s with

the first reports of noninvasive evaluation of resting myocardial blood flow. Since that time, there

have been major advances in the technical ability to image cardiac physiology and

pathophysiology, including that of myocardial blood flow, myocardial metabolism, and

ventricular function.

Standard References :

1. McPhee SJ, Papadakis MA. Current Medical Diagnosis & Treatment. 47th ed. New York:

Lange Medical Book`s/The McGraw-Hill Companies, 2008.p.

2. Roentgen Signs in Diagnostic Imaging Isadore Meschan.

SELF DIRECTING LEARNING

Basic knowledge that must be known:

1. History taking

2. Common cardiological symptom and consultation

3. Investigation in Cardiology

LEARNING TASK I: Investigation in pediatric cardiology

1. When you suppose that the patient may be suffering from CHD?

2. Which one who is the most sensitive sign and applied as the best screening of CHD?

3. What is the most specific sign of CHD?

4. Please explain sign of the left and right heart hypertrophy by inspection and palpitation!

5. Please mention (a) diagnostic tool(s) in pediatric cardiology!

6. When complete blood count should be performed?

7. Please describe the site of classic heart sound and characteristics of that!

LEARNING TASK II: Investigation in cardiology

1. Please explain the symptoms in patients with cardiovascular disease!

2. What are the diagnostic tools to confirm patients with Ischaemic Heart Disease?

3. What is the benefit of 24-hour electrocardiogram?

4. What is the objective of performing stress testing (treadmill test)?

LEARNING TASK III: RADIOLOGY

1. What are the basic projections for cardiac radiography?

2. Explain normal anatomy of the heart on the chest x-ray!

3. Explain cardiac enlargement on the chest x-ray!

SELF ASSESSMENT :

1. Please explain the symptoms in patients with cardiovascular disease!

2. What are the diagnostic tools to confirm patients with Ischaemic Heart Disease?

3. What is the benefit of 24-hour electrocardiogram?

4. What is the objective of performing stress testing (treadmill test)?

5. Please describe the chest x-ray finding in VSD!

6. Differentiated between LVH and RVH on chest x-ray!

7. Explain HHD on the chest x-ray!

8. Explain tetralogy of Fallot on the chest x-ray!

Page 26: STUDY GUIDE THE CARDIOVASCULAR SYSTEM AND DISORDERS · study guide the cardiovascular system and disorders ... study guide the cardiovascular system and disorders ... (ekg): pemasangan

Study Guide Cardiovascular System and Disorders

Department of Medical Education - Faculty of Medicine - Universitas Udayana, 2018

26

LECTURE 2

CARDIOVASCULAR PHARMACOLOGY

dr. Agung Nova Mahendra, M.Sc.

Department of Pharmacology and Therapy

AIMS

The students are expected to know and comprehend the basics and principles of blood

pressure regulation, pathophysiology of angina pectoris, cardiac performance modulation,

arrhythmia mechanisms, and able to apply these knowledges into rational pharmacotherapy of

primary hypertension (HT), angina pectoris, acute and chronic heart failure (HF), and selected

arrhythmias.

LEARNING OUTCOMES

At the end of the lectures, the students are able to:

1. Comprehend and describe therapeutic strategies in HT, angina, HF, and arrhythmias

pharmacy management.

2. Describe important pharmacological features of drugs used in HT, angina, HF, and

arrhythmias.

3. Comprehend clinical applications of drugs used in HT, angina, HF, and arrhythmias.

4. Increase awareness of important or common toxicities and DDIs of drugs used in HT,

angina, HF, and arrhythmias.

CURRICULUM CONTENTS

1. Pathophysiology-based therapeutic strategies in HT, angina, HF, and arrhythmias

pharmacy management.

2. PD and PK of drugs used in HT, angina, HF, and arrhythmias.

3. Clinical applications of drugs used in HT, angina, HF, and arrhythmias.

4. Toxicities and DDIs of drugs used in HT, angina, HF, and arrhythmias.

ABSTRACT

Cardiovascular disorders are leading causes of hospitalization and mortality in the world (Liu et

al., 2010). Among these disorders, hypertension (HT), angina, heart failure (HF), and

arrhythmias are commonly seen in emergency and general clinical practice. Thus, basic

pharmacological knowledge of drugs used in these conditions is the essential pillar of rational

pharmacotherapy to save many lives and improve cardiovascular system health.

STANDARD REFERENCE

Katzung, BG, Trevor AJ. 2015. Basic and Clinical Pharmacology Thirteenth Edition. McGraw-

Hill Education: New York.

SELF-DIRECTED LEARNING

Basic knowledge as the prerequisites of the lecture are:

1. Blood pressure regulation

2. Pathophysiology of angina pectoris

3. Control of normal cardiac contractility

4. Pathophysiology of HF & cardiac performance.

Page 27: STUDY GUIDE THE CARDIOVASCULAR SYSTEM AND DISORDERS · study guide the cardiovascular system and disorders ... study guide the cardiovascular system and disorders ... (ekg): pemasangan

Study Guide Cardiovascular System and Disorders

Department of Medical Education - Faculty of Medicine - Universitas Udayana, 2018

27

SCENARIO

Mr. C, 53 years-old, accompanied by his children to an ER, complaining about gasping or

breath every night for about 1-2 weeks. The symptom starts at night and causes him to awake

after several hours of sleep. This symptom is accompanied by an anxious feeling of being

suffocated and persists longer than 30 minutes. History of asthmatic attacks is denied, but he

admits that he is a binge “tuak” and “arak” drinker, shows abnormal lipid profile (during his last

MCU about 3 weeks before), and has a familial history of hypertension and heart attack. After

anamnesis and a set of physical and laboratory examinations, diagnosis of heart failure is

established.

LEARNING TASKS

1. Describe the therapeutic strategies in HT, angina, and HF pharmacy management!

2. What are the classes of drugs used in HT, angina, HF?

3. Describe the classes of antiarrhythmic drugs and their pharmacodynamic characteristics!

4. What are the clinical uses of each class of drugs used in HT, angina, HF, and

arrhythmias?

5. Describe the important/common toxicities & potential drug-drug interactions (DDIs) of

drugs used in HT and angina!

SELF-ASSESSMENT

1. What are the differences between ACEIs and ARBs?

2. What is the rationale of combining β-blocker with nitrate in the therapy of angina?

3. What are the targets of action of amiodarone?

4. Name one anti-HF drug that acts selectively on cardiac β1 adrenoceptor!

Page 28: STUDY GUIDE THE CARDIOVASCULAR SYSTEM AND DISORDERS · study guide the cardiovascular system and disorders ... study guide the cardiovascular system and disorders ... (ekg): pemasangan

Study Guide Cardiovascular System and Disorders

Department of Medical Education - Faculty of Medicine - Universitas Udayana, 2018

28

LECTURE 3

PATHOLOGICAL ANATOMY OF CARDIOVASCULAR SYSTEM: ANOMALIC

FORMATION OF THE HEART AND GREAT VESSELS

dr. Ni Putu Ekawati, Sp.PA, M.Repro.

Department of Pathology Anatomy

AIMS:

Describe the basic principles underlying the formation of anomalies of the heart and great

vessels.

LEARNING OUTCOME:

1. Can describe the basic principles underlying the formation of anomalies of the heart and

its implications

2. Can describe the basic principles underlying the formation of anomalies of the great

vessels and its implications

CURRICULUM CONTENT:

1. Embryology of the heart

2. Embryology of the great vessels.

ABSTRACT

Septal defects are only problematic when the shunt flows from right-to-left. Anomalies of

interventricular septum (VSD) usually happens at the upper membranous portion that composed

of connective tissue continuous with the annulus fibrosus. A small VSD may result in an

inconsequential left-to-right shunt.

In the presence of pulmonary stenosis, a VSD produces a right-to-left shunt with cyanosis and

the blue-baby syndrome. A large VSD is a principal factor in Tetralogy of Fallot.

Atrial septal defects (ASD) are most common in the vicinity of the fossa ovalis. Septum

secundum defects, the typical patent foramen ovale, account for 10-15% of all cardiac

anomalies. Normal left atrial pressure is slightly greater than right atrial pressure, a left-to-right

shunt occurs through an open ASD, oxygenated blood from the left side of the heart is shunted

to the right side, thus not associated with cyanosis. An ASD is usually compatible with normal

life, except at an extreme exercise, cardiac disease, or pulmonary disease alter chamber

pressures, a right-to-left shunt will produce cyanosis.

Patent ductus arteriosus (PDA) is a persistence of the fetal connection (ductus arteriosus)

between the aorta and pulmonary artery after birth, resulting in a left-to-right shunt. Symptoms

may include failure to thrive, poor feeding, tachycardia, and tachypnea. A continuous machine-

like murmur in the upper left sternal border is common. Diagnosis is by echocardiography.

Standard References:

1. Moore KL, Agur AMR: Essential Clinical Anatomy, 3rd ed. Philadelphia, Lippincott &

Wilkins, 2007. p. 91, 93, 94

2. Sadler TW: Langman’s Medical Embryology, 10th ed. Philadelphia, Lippincott & Wilkins,

2006. p. 167-178, 184

Page 29: STUDY GUIDE THE CARDIOVASCULAR SYSTEM AND DISORDERS · study guide the cardiovascular system and disorders ... study guide the cardiovascular system and disorders ... (ekg): pemasangan

Study Guide Cardiovascular System and Disorders

Department of Medical Education - Faculty of Medicine - Universitas Udayana, 2018

29

SELF DIRECTING LEARNING

Basic knowledge that must be known:

1. Embryology of the heart

2. Embryology of the great vessel.

SCENARIO: CASE 1

This baby aged 4 months has been known to have a cardiac murmur since birth. He was born 8

weeks prematurely and developed respiratory distress requiring high oxygen concentration for

the first week. Since then he has feed satisfactorily but height and weight growth have been

poor even allowing for prematurity. The diagnosis after examination and investigations: Patent

Ductus Arteriosus (PDA).

LEARNING TASK I

1. What factors in the history were of possible importance in causing the ductus remains

open? Why there is no cyanosis in this case?

2. Why is there no cyanosis in this case?

3. Why was the heart murmur audible in diastole as well as systole?

4. Why is there evidence of left ventricular hypertrophy and not right ventricular

hypertrophy?

5. Why is there pulmonary congestion?

6. Why the shunt from the aorta to pulmonary artery and not vice versa?

7. After an operation to close the PDA, why is there a risk of the patient becoming hoarse?

CASE 2

This 13-year-old girl was recently found to have a cardiac murmur. She has been generally

healthy with good growth, but on questioning her mother admitted she has noticed that girl

tends to tire easily with exercise. The diagnosis of examination and investigations: Atrial Septal

Defect (ASD).

LEARNING TASK II:

1. Why is there a mild chest deformity with a bulge in the thoracic cage to the left of the

sternum?

2. Why is there no cyanosis?

3. Why does the right ventricle carry a volume load in A.S.D., while it is the left ventricle in

PDA? Both are a left-to-right shunt. Consider the appropriate anatomy involved.

4. Why is there a systolic murmur over the pulmonary valve and a diastolic murmur over

the tricuspid valve?

CASE 3 :

A 2-year-old boy was admitted to the hospital for evaluation of a heart murmur previously detect

at birth. He was less active than other children his age, but although over-exertion was followed

frequently by cyanosis of the lips and nails, there was no history of unconsciousness. Initial

examination revealed a thin, physically retarded, cyanotic child with no respiratory difficulty.

There was moderate clubbing of the fingers. A harsh systolic murmur was maximal over the

mid-precordial area. The first heart sound was normal while the second was single, distinct and

Page 30: STUDY GUIDE THE CARDIOVASCULAR SYSTEM AND DISORDERS · study guide the cardiovascular system and disorders ... study guide the cardiovascular system and disorders ... (ekg): pemasangan

Study Guide Cardiovascular System and Disorders

Department of Medical Education - Faculty of Medicine - Universitas Udayana, 2018

30

loud.The lungs were clear. X-ray showed a normal sized heart dominated by a boot-shaped

right ventricular outflow tract. Diagnosis of Tetralogy of Fallot.

LEARNING TASK III:

1. Mention the cardiac abnormality you found in this case!

2. What is the basic defect of this heart malformation?

3. What is the most important abnormality causing cyanosis?

4. Why was he less active than other children his age?

5. Why is he revealed thin and physically retarded?

6. Why was there clubbing of his fingers?

SELF-ASSESSMENT :

1. Describe the principal normal development of the heart and pericardium!

2. Named the most common congenital anomalies of the heart with their clinical

implications!

3. Describe the abnormities, the hemodynamic changes, the incidence and the clinical

implications in general population of ventricular septal defect (VSD), Tetralogy of Fallot,

and atrial septal defect (ASD)!

4. Describe the blood flow before and after birth and changes occur in the vascular system

after birth!

Page 31: STUDY GUIDE THE CARDIOVASCULAR SYSTEM AND DISORDERS · study guide the cardiovascular system and disorders ... study guide the cardiovascular system and disorders ... (ekg): pemasangan

Study Guide Cardiovascular System and Disorders

Department of Medical Education - Faculty of Medicine - Universitas Udayana, 2018

31

LECTURE 4

LABORATORY EXAMINATION: CARDIAC MARKERS

Dr. dr. Wiradewi, Sp.PK

Department of Clinical Pathology

AIMS:

Can describe various cardiac markers.

LEARNING OUTCOME:

Able to interpret cardiac marker results.

CURRICULUM CONTENTS:

1. Different cardiac markers

2. Mechanisms of each cardiac marker

3. Cardiac marker results

ABSTRACT :

Although many patients with an acute myocardial infarction (AMI) will present with the classic

history of severe chest pain and have ECG findings of ST elevation and pathologic Q waves,

approximately 25% of patients will not. Approximately 50% of patients who come through the

emergency room with a myocardial infarction have nondiagnostic ECGs. It is for this reason that

when a patient is suspected of having myocardial damage, in addition to close hemodynamic

monitoring, a good physical exam, and serial EKGs, a set of cardiac markers should be

obtained.

The cardiac markers include myoglobin, creatine kinase (CK), the MB fraction of creatine kinase

(CK-MB), aspartate aminotransferase (AST), lactate dehydrogenase (LDH) and troponin. These

are intracellular cardiac markers normally detectable in the blood at very low levels. As the

myocytes become necrotic, the cardiac markers will leak out of these cells and ultimately into

the vasculature where they can be detected by routine laboratory testing.

Myoglobin is an oxygen-binding protein that is released from myocardial cells when they are

injured. It can be detected 1-4 hours after the insult and peaks at 4-12 hours. Creatine kinase is

another protein that is released from the myocardial cell when it is damaged. Its activity is

greatest in striated muscle, brain and heart tissue. CK-MB levels are detectable at 4-6 hours

postinjury, peak at 24 hours. Aspartate aminotransferase is an enzyme that is also released with

myocardial infarction. Like CK, in acute MI, AST rises to a maximum at 24 hours and declines to

normal levels at about 48 hours postinfarction. Lactate dehydrogenase is an enzyme that is

released during myocardial infarction. AST and LDH are not specific to cardiac diseases.

Troponin is the most specific of the commercially available cardiac markers. Troponin can be

measured as early as 3 hours after the onset of chest pain.

Page 32: STUDY GUIDE THE CARDIOVASCULAR SYSTEM AND DISORDERS · study guide the cardiovascular system and disorders ... study guide the cardiovascular system and disorders ... (ekg): pemasangan

Study Guide Cardiovascular System and Disorders

Department of Medical Education - Faculty of Medicine - Universitas Udayana, 2018

32

STANDARD REFERENCES:

1. Burtis CA, Bruns DE: Tietz Fundamentals of Clinical Chemistry and Molecular

Diagnostics, 7th ed. St.Louis, MO: Saunders/Elsevier, 2014.

2. Bishop ML, Fody EP, Schoeff LE: Clinical Chemistry: Principles, Techniques, And

Correlations, 7th ed. Philadelphia: Lippincott Williams & Wilkins, 2013.

3. Desai SP, Isa-Pratt S: Clinician’s Guide to Laboratory Medicine: A Practical Approach,

3rd ed. Michigan: Lexi-Comp, 2004.

SELF DIRECTING LEARNING

Basic knowledge that must be known:

1. Types of cardiac marker.

2. Mechanisms of each cardiac marker.

LEARNING TASKS

1. A patient came to the emergency room with complaints of chest pain, like being pressed

by a heavy object. The pain is intermittent and reduced if the patient is resting. What

kind of laboratory tests are to be proposed? If the laboratory results are normal and the

ECG was also normal. What is the diagnosis of this patient?

2. A patient came to the emergency room with complaints of chest pain since 5 hours ago.

Patients never feel the same complaint before. Patients had a history of gastric ulcer.

The laboratory results are as follows :

Myoglobin: increased

CK-MB: normal

Troponin normal

What is the diagnosis of this patient?

3. A patient came to the emergency room with complaints of chest pain, like being pressed

by a heavy object. Patients can not say exactly when the pain was beginning. Laboratory

results are as follows:

Myoglobin: normal

CK-MB: increased

Troponin: increased

What is the diagnosis of this patient? Can you estimate when the cell damage occurs?

SELF ASSESSMENTS

1. What are cardiac markers?

2. What other conditions can cause an increase in CK-MB?

3. How does troponin compare with CK?

4. What role do these cardiac markers play in monitoring reperfusion following thrombolytic

therapy?

Page 33: STUDY GUIDE THE CARDIOVASCULAR SYSTEM AND DISORDERS · study guide the cardiovascular system and disorders ... study guide the cardiovascular system and disorders ... (ekg): pemasangan

Study Guide Cardiovascular System and Disorders

Department of Medical Education - Faculty of Medicine - Universitas Udayana, 2018

33

LECTURE 5

REHABILITATION ON CARDIOVASCULAR DISEASE PATIENTS

dr. Luh Kamiati, Sp.KFR

Department of Rehabilitation Medic

AIMS:

Describe cardiac rehabilitation on patient with Cardiovascular Disease.

LEARNING OUTCOME:

Able to describe cardiac rehabilitation in patient with cardiovascular disease.

CURRICULUM CONTENTS:

Cardiac rehabilitation in patient with cardiovascular disease.

ABSTRACT:

Cardiac rehabilitation is a multidisciplinary program of education and exercise established to

assist an individual with heart disease in achieving optimal physical, psychological and

functional status within the limits of the diseased.

The basic goal of cardiac rehabilitation is to restore and improve cardiac function, reduce

disability, identify and cardiac risk factors, increased cardiac conditioning. Cardiac rehabilitation

programs consist primary prevention (education, behavior modification), secondary prevention,

and exercise program.

Cardiac rehabilitation outcomes that can be expected the decreased length of hospital stay,

more rapid, more complete resumption of usual activities, self-confident, less psychological

distress, and fewer readmissions.

Standard References :

1. McPhee SJ, Papadakis MA. Current Medical Diagnosis & Treatment. 47thed. New York:

Lange Medical Book`s/The McGraw-Hill Companies, 2008.p. 287-299, 351-358; 398-

416, 360-363; 1241-1246

2. Garrison SJ: Hand Book of Physical Medicine and Rehabilitation, 2nded, 2003, p. 86

3. Bartels MN: Cardiac Rehabilitation in Grant Cooper: Essential Physical Medicine and

Rehabilitation, 2006, p. 119.

SCENARIO

CASE:

A 60 years old man admitted to the emergency room because of chest pain. He was diagnosed

with acute heart failure caused by coronary artery disease. He is now stabilized transferred to

the cardiac ward.

Page 34: STUDY GUIDE THE CARDIOVASCULAR SYSTEM AND DISORDERS · study guide the cardiovascular system and disorders ... study guide the cardiovascular system and disorders ... (ekg): pemasangan

Study Guide Cardiovascular System and Disorders

Department of Medical Education - Faculty of Medicine - Universitas Udayana, 2018

34

LEARNING TASK:

1. Mention the definition of cardiovascular rehabilitation

2. Explain the objective of cardiovascular rehabilitation

3. Mention the contraindication exercise therapy

4. Explain the beneficial effect of exercise therapy

5. Explain stages of cardiovascular rehabilitation MI

6. Mention effect of exercise to CHF.

SELF DIRECTING LEARNING and SELF-ASSESSMENT

1. Mention definition of cardiovascular rehabilitation

2. Explain the objective of cardiovascular rehabilitation

3. Mention the contraindication exercise therapy

4. Explain the beneficial effect of exercise therapy

5. Explain stages of cardiovascular rehabilitation MI

6. Mention effect of exercise to CHF.

Page 35: STUDY GUIDE THE CARDIOVASCULAR SYSTEM AND DISORDERS · study guide the cardiovascular system and disorders ... study guide the cardiovascular system and disorders ... (ekg): pemasangan

Study Guide Cardiovascular System and Disorders

Department of Medical Education - Faculty of Medicine - Universitas Udayana, 2018

35

LECTURE 6

ACUTE CORONARY SYNDROME

dr. I Made Junior Rina Artha, Sp JP (K), FIHA, FAsCC

Department of Cardiology and Vascular Medicine

AIMS:

Describe to diagnose and manage Acute Coronary Syndromes (ACS).

LEARNING OUTCOME:

1. Defining acute coronary syndromes (ACS),

2. Examining ACS Modifiable and non-modifiable risk factors,

3. Describing the pathophysiology of unstable angina (UA), non ST-elevation myocardial

infarction (NSTEMI), and ST-elevation myocardial infarction STEMI).

CURRICULUM CONTENTS:

Pathogenesis of atherothrombosis.

Risk factors for Acute Coronary Syndromes.

Clinical spectrum of Acute Coronary Syndromes.

Interpret laboratory of Acute Coronary Syndromes.

Interpret diagnostic tools for Acute Coronary Syndromes.

Management and its prognosis of Acute Coronary Syndromes.

Post ACS medical rehabilitation and its rehabilitation.

ABSTRACT:

Coronary artery disease (CAD) is one of the most important causes of premature death in the

developed world, as well in Indonesia. CAD is regarded as a leading cause of mortality in

Province of East Java and Bali, based on the National Household Health Survey in 1995. Its

proportion was reported to be 24.5% of all-cause mortality, and its proportion has been

significantly increased since the last 10 years in Indonesia (SKRT, 1995 and accounts for one in

seven deaths in the US, killing over 360,000 people a year. For an individual at 40 years old, the

lifetime risk of developing CHD is 49% in men and 32% in women; for those reaching age 70

years, the lifetime risk is 35% in men and 24% in women. Coronary atherosclerosis, the basic

pathogenesis of this disease, is associated with many risk factors such as cigarette smoking,

hyperlipidemia, family history, hypertension and diabetes mellitus.

Atherosclerosis is a chronic process initiated by lipid deposition and vascular wall injury that

causes increased endothelial permeability, inflammation and recruitment of monocytes and

leucocytes. These cells accumulate oxidized lipids to form macrophages and foam cells, and

lead to the formation of ‘fatty streak’ and then ‘atheroma’. Eventually, all these processes

becomes ‘atherosclerotic plaque’. ACS caused by clots formed in the heart, once the plaque

ruptured, it bleeds and initiates the release of platelets through the clotting process. There is

thrombosis/clot formation which further occludes the diameter of the coronary artery and

decreases blood supply to the heart. Chronic stable angina is caused by atheroma obstructing

coronary artery lumen by more than 70%. Acute coronary syndromes (ACS = unstable angina

and myocardial infarction) arise when an atherosclerotic plaque becomes unstable and either

Page 36: STUDY GUIDE THE CARDIOVASCULAR SYSTEM AND DISORDERS · study guide the cardiovascular system and disorders ... study guide the cardiovascular system and disorders ... (ekg): pemasangan

Study Guide Cardiovascular System and Disorders

Department of Medical Education - Faculty of Medicine - Universitas Udayana, 2018

36

ruptures or are eroded. The complicated plaque is a nidus for thrombus formation and may lead

to vessel occlusion.

The acute coronary syndromes encompass a spectrum of unstable coronary artery disease that

includes unstable angina and myocardial infarction (ST-segment elevation myocardial infarction

and non-ST segment elevation myocardial infarction). The history, electrocardiogram, and

cardiac markers determine the presence and the type of ACS. Patients with an acute coronary

syndrome usually Presenting symptoms when plaque continues to occlude the coronary artery

and/or the coronary artery is contracted, blood carrying the necessary oxygen and nutrients

cannot pass through, thus causing ischemia. Patients present with right or left sided chest pain,

discomfort, pressure or tightness around the chest more than 20 minutes, other symptoms

include pain or discomfort in the shoulder, neck, jaw or back, dyspnea, nausea, vertigo, and

diaphoresis. present with prolonged anginal symptoms that occur at rest. The electrocardiogram

will often show evidence of ischemia that classically takes the form of ST segment shifts, T-

wave inversion, and new bundle branch block. Cardiac enzymes and markers are the principal

determinants that defines the category of the acute coronary syndrome. Patients should be

given analgesia, oxygen and transferred to intensive coronary care unit. Treatments consist of

aspirin, clopidogrel, low-molecular-weight heparin, beta-blockers and intravenous nitrate

infusion. Where available, percutaneous coronary intervention (PCI) is the treatment of choice.

Thrombolytic therapy is an effective alternative.

SELF DIRECTING LEARNING

Basic knowledge that must be known:

1. Risk factors for acute coronary syndromes

2. Clinical spectrum of acute coronary syndromes

3. Interpret laboratory of acute coronary syndromes

4. Interpret diagnostic tools for acute coronary syndromes

5. Management and its prognosis of acute coronary syndromes

6. Post ACS medical rehabilitation and its rehabilitation.

References:

1. Mann, DL et al. Braunwald’s Heart Disease, 10th ed. Philadelphia, Elsevier Saunders,

2015.

2. Ibanez B, et al. 217 ESC Guidelines for management of Acute Myocardial Infarction in

Patients Presenting with ST-Segment Elevation. Eur Heart J. 2017:39,119-177.

SCENARIO 1:

CASE (Acute Coronary syndrome):

A 56-year old male came to Emergency Room due to severe chest pain. The pain was felt since

6-hours prior to admission, it was a heavy sensation and radiating to left arm and jaw, not

relieved by rest, accompanied by shortness of breath since 5 hours PTA that is relieved by

sitting position. He is suffered from uncontrolled DM and Hypertension since 7 years ago. He

appeared severely ill and dyspnea. The blood pressure was 170/90 mmHg; pulse rate was 110

beats-per-minute, regular. There were rales on both lung fields ECG showed:

Page 37: STUDY GUIDE THE CARDIOVASCULAR SYSTEM AND DISORDERS · study guide the cardiovascular system and disorders ... study guide the cardiovascular system and disorders ... (ekg): pemasangan

Study Guide Cardiovascular System and Disorders

Department of Medical Education - Faculty of Medicine - Universitas Udayana, 2018

37

LEARNING TASK:

1. What is the most likely diagnosis?

2. What do the next procedure you plan?

3. What is your initial treatment?

4. What is the definitive therapy for this case?

5. What is the could be a complication for this case?

SELF-ASSESSMENT:

1. Please explain the risk factors of acute coronary syndromes!

2. What are the complications of acute myocardial infarction?

3. What is the treatment of choice in ST-elevation myocardial infarction?

4. Please describe the indication and the benefit of CABG (coronary artery bypass graft)!

Page 38: STUDY GUIDE THE CARDIOVASCULAR SYSTEM AND DISORDERS · study guide the cardiovascular system and disorders ... study guide the cardiovascular system and disorders ... (ekg): pemasangan

Study Guide Cardiovascular System and Disorders

Department of Medical Education - Faculty of Medicine - Universitas Udayana, 2018

38

LECTURE 7

Stable Coronary Artery Disease

dr. Hendy Wirawan, M.Biomed, Sp JP, FIHA

Department of Cardiology and Vascular Medicine

AIMS:

Describe diagnostic approach and management Stable Coronary Artery Disease (SCAD).

LEARNING OUTCOME:

Be able to describe diagnostic approach and management Stable Coronary Artery Disease.

CURRICULUM CONTENT:

Pathogenesis of atherothrombosis.

Risk factors for Stable Coronary Artery Disease.

Clinical spectrum of Stable Coronary Artery Disease.

Interpret diagnostic tools for Stable Coronary Artery Disease.

Management and its prognosis of Stable Coronary Artery Disease.

ABSTRACT:

Stable Coronary artery disease (SCAD) is most commonly caused by atheromatous plaque that

obstructs or gradually narrows one or more of the epicardial coronary arteries. Other

contributors, such as endothelial dysfunction, microvascular disease, and vasospasm may also

exist alone or in combination with coronary atherosclerosis and may be the dominant cause of

myocardial ischemia in some patients. Thus the concept that coronary artery disease is

synonymous with obstructive coronary atherosclerosis and ischemic heart disease (IHD).

Coronary artery disease (CAD) is a major cause of death and disability in developed countries.

Although the mortality for this condition has gradually declined over the last decades in western

countries, it still causes about one-third of all deaths in people older than 35 years. In

Indonesia, CAD became the second highest cause of death after stroke, which amounted to

12.4% based on System Registration Sample Survey (SRS). It is estimated that 15.400.000

Americans have CAD, 7.800.000 of whom have angina pectoris and 7.600.000 have had MI.

The basic pathogenesis of the disease is coronary atherosclerosis, a chronic process to form

atherosclerotic plaque. Initiated by lipid deposition and vascular wall injury that causes

increased endothelial permeability, inflammation and recruitment of monocytes and leucocytes.

These cells accumulate oxidized lipids to form macrophages and foam cells, and lead to the

formation of ‘fatty streak’ and then atheroma. Over time, plaque hardens and narrows the

coronary artery, this limits the flow of oxygen-rich blood to the myocardium. Conventional risk

factors that increase the progression of atherosclerosis are hypertension, hypercholesterolemia,

diabetes, sedentary lifestyle, obesity, smoking and family history.

Clinical manifestation of CAD can be either stable angina pectoris/ stable CAD and unstable

angina pectoris/ acute coronary syndrome (ACS). SCAD is generally characterized by episodes

of reversible myocardial demand/supply mismatch, related to ischemia or hypoxia, which are

usually inducible by exercise, emotion or other stress and reproducible—but, which may also be

occurring spontaneously. Such episodes of ischemia/hypoxia are commonly associated with

Page 39: STUDY GUIDE THE CARDIOVASCULAR SYSTEM AND DISORDERS · study guide the cardiovascular system and disorders ... study guide the cardiovascular system and disorders ... (ekg): pemasangan

Study Guide Cardiovascular System and Disorders

Department of Medical Education - Faculty of Medicine - Universitas Udayana, 2018

39

transient chest discomfort (angina pectoris). Whereas ACS almost always presents with a

symptom, such as unstable angina, and is frequently associated with myocardial infarction (MI).

The diagnosis and assessment of SCAD involve clinical evaluation, including identifying

significant dyslipidemia, hyperglycemia or other biochemical risk factors and specific cardiac

investigations such as stress testing or coronary imaging. A careful history remains the

cornerstone of the diagnosis of chest pain. The characteristics of discomfort-related to

myocardial ischemia (angina pectoris) may be divided into four categories: location, character,

duration, and relationship to exertion and other exacerbating or relieving factors. The discomfort

caused by myocardial ischemia is usually located in the chest, near the sternum, but may be felt

anywhere from the epigastrium to the lower jaw or teeth, between the shoulder blades or in

either arm to the wrist and fingers. The discomfort is often described as pressure, tightness or

heaviness; sometimes strangling, constricting or burning. The duration of the discomfort is

brief—no more than 10 min in the majority of cases and more commonly even minutes or less—

but chest pain lasting for seconds is unlikely to be due to angina. An important characteristic is a

relationship to exercise, specific activities or emotional stress. Symptoms classically appear or

become more severe with increased levels of exertion and rapidly disappear within a few

minutes when these causal factors abate.

Basic (first-line) testing in patients with suspected SCAD includes standard laboratory

biochemical testing, a resting ECG, possibly ambulatory ECG monitoring (if there is a clinical

suspicion that symptoms may be associated with a paroxysmal arrhythmia), resting

echocardiography and, in selected patients, a chest X-ray (CXR). Such testing can be done on

an outpatient basis. The aim of the management of SCAD is to reduce symptoms and improve

prognosis. The management of CAD patients encompasses lifestyle modification, control of

CAD risk factors, evidence-based pharmacological therapy and patient education.

SELF DIRECTING LEARNING

Basic knowledge that must be known:

1. Risk factors for Ischaemic Heart Disease

2. Clinical spectrum of Ischaemic Heart Disease

3. Interpret diagnostic tools for Ischaemic Heart Disease

4. Management and its prognosis of Ischaemic Heart Disease.

References:

1. Mann, DL et all. Braunwald’s Heart Disease, 10th ed. Philadelphia, Elsevier Saunders,

2015.

2. Montalescot G. Et al. 2013 ESC Guidelines on The Management of Stable Coronary

Artery Disease. Eur H J. 2013:34,2949-3003.

Page 40: STUDY GUIDE THE CARDIOVASCULAR SYSTEM AND DISORDERS · study guide the cardiovascular system and disorders ... study guide the cardiovascular system and disorders ... (ekg): pemasangan

Study Guide Cardiovascular System and Disorders

Department of Medical Education - Faculty of Medicine - Universitas Udayana, 2018

40

SCENARIO :

CASE (Ischemic Heart Disease) :

A 68-year-old man presents to his new primary care provider for an initial patient visit. His sole

active complaint is mild retrosternal chest pressure that he experiences only after significant

exertion, and the discomfort resolves within minutes of cessation of the provoking activity. The

pattern and the severity of his symptoms have not changed significantly over the last 3 years,

and he denies any episodes of chest pain at rest, dizziness, syncope, dyspnea, palpitations, or

lower extremity edema. Other pertinent medical history includes hypercholesterolemia and long-

standing hypertension. On examination, the patient is comfortable and in no acute distress. The

heart rate is 90 bpm and regular, and the blood pressure 155/85 mmHg. There are no murmurs,

rubs, gallops or clicks on cardiac auscultation and the second heart sound is physiologically

split. A resting ECG is normal.

LEARNING TASK :

1. What is the most likely diagnosis?

2. What is the best next diagnostic step?

3. What is the best next step in therapy?

SELF-ASSESSMENT :

1. Please explain the risk factors for coronary artery disease!

2. Please describe laboratory recommendation for assessment stable angina!

3. What is the noninvasive and invasive recommendation for assessment stable angina?

4. What is the treatment of choice for Stable angina pectoris?

5. Please describe the indication and the benefit of revascularization!

Page 41: STUDY GUIDE THE CARDIOVASCULAR SYSTEM AND DISORDERS · study guide the cardiovascular system and disorders ... study guide the cardiovascular system and disorders ... (ekg): pemasangan

Study Guide Cardiovascular System and Disorders

Department of Medical Education - Faculty of Medicine - Universitas Udayana, 2018

41

LECTURE 8

HYPERTENSION

dr. AA Ayu Dwi Adelia Yasmin, M.Biomed, SpJP, FIHA

Department of Cardiology and Vascular Medicine

AIMS:

Describe diagnostic approach and management of Hypertension.

LEARNING OUTCOME:

Be able to describe diagnostic approach and management of Hypertension.

CURRICULUM CONTENTS:

1. Etiology and pathophysiology of Hypertension

2. Clinical criteria and classification of Hypertension

3. Diagnostic approach to Hypertension

4. Management and prognosis of Hypertension

ABSTRACT:

Hypertension is the most common condition seen in primary care and leads to myocardial

infarction, stroke, renal failure, and death if not detected early and treated appropriately.

Hypertension is defined as values ≥140 mmHg Systolic Blood Pressure (SBP) and/or ≥90

mmHg Diastolic Blood Pressure (DBP), based on the evidence from Randomized Controlled

Trials that in patients with these BP values treatment-induced BP reductions are beneficial. The

same classification is used in young, middle-aged and elderly subjects. Overall the prevalence

of hypertension appears to be around 30–45% of the general population, with a steep increase

with aging.

Based on recommendations of the Seventh Report of the Joint National Committee on

Prevention, Detection, Evaluation, and Treatment of High Blood Pressure (JNC-7), the

classification of BP for adults aged 18 years or older has been as follows :

1. Normal: Systolic lower than 120 mm Hg, diastolic lower than 80 mm Hg

2. Prehypertension: Systolic 120-139 mm Hg, diastolic 80-89 mm Hg

3. Stage 1: Systolic 140-159 mm Hg, diastolic 90-99 mm Hg

4. Stage 2: Systolic 160 mm Hg or greater, diastolic 100 mm Hg or greater

Hypertension may be primary, which may develop as a result of environmental or genetic

causes, or secondary, which has multiple etiologies, including renal, vascular, and endocrine

causes. Primary or essential hypertension accounts for 90-95% of adult cases, and secondary

hypertension accounts for 2-10% of cases. Many guidelines exist for the management of

hypertension. Most groups recommend lifestyle modification as the first step in managing

hypertension. If lifestyle modifications are insufficient to achieve the goal BP, there are several

drug options for treating and managing hypertension. Most individuals diagnosed with

hypertension will have increased blood pressure (BP) as they age. Untreated hypertension is

notorious for increasing the risk of mortality and is often described as a silent killer. Mild to

moderate hypertension, if left untreated, may be associated with a risk of atherosclerotic

disease in 30% of people and organ damage in 50% of people within 8-10 years after onset.

Page 42: STUDY GUIDE THE CARDIOVASCULAR SYSTEM AND DISORDERS · study guide the cardiovascular system and disorders ... study guide the cardiovascular system and disorders ... (ekg): pemasangan

Study Guide Cardiovascular System and Disorders

Department of Medical Education - Faculty of Medicine - Universitas Udayana, 2018

42

Standard References:

1. Chobanian A, et al. Seventh Report of the Joint National Committee On Prevention,

Detection, Evaluation, And Treatment Of High Blood Pressure. Hypertension.

2003;42:1206–1252.

2. Mancia G, et al. 2013 ESH/ESC Guidelines for the Management of Arterial

Hypertension. European Heart Journal. 2013;34:2159-2219.

3. James PA, et al. 2014 Evidence-Based Guideline for the Management of High Blood

Pressure in Adults. Report From the Panel Members Appointed to the Eighth Joint

National Committee (JNC 8). 2014;311(5):507-520.

SCENARIO CASE:

A 60-year-old man decides it is time to have a health check and attends his local GP. As part of

the assessment, the practice nurse takes his blood pressure, which is recorded as 170/90

mmHg. He is 170 m tall, weighs 80 kg and has no past medical history. His basic physical

examination is unremarkable with no audible murmurs on cardiac auscultation. He has no

history of cardiac symptoms or diabetes. His 80-year-old mother is currently on blood pressure

tablets and his father died at the age of 74 years from a heart attack. He stopped smoking 20

cigarettes a day 20 years ago. He drinks 2–3 units of alcohol most days, sometimes more at

weekends. His only exercise is walking the dog. He says he eats well and usually adds salt to

his meals after they are cooked

LEARNING TASK:

1. How do you interpret the patient’s blood pressure reading?

2. What is your advice and management plan for this patient?

3. He asks you why he has high blood pressure. What do you tell him and would you

perform any additional tests or examinations?

4. He is reluctant to have any treatment. What are the potential consequences of

uncontrolled hypertension? And what are your suggestions to him?

5. What would you prescribe him?

6. What potential are side-effects there from his medication?

SELF-ASSESSMENT:

1. What are the lifestyle modification recommendations for a patient with hypertension

based on guidelines?

2. Please explain the definition of white coat hypertension and masked hypertension, and

how can we diagnose it?

3. What are the side effects of ACE Inhibitor, Diuretic, and Calcium channel blocker?

Page 43: STUDY GUIDE THE CARDIOVASCULAR SYSTEM AND DISORDERS · study guide the cardiovascular system and disorders ... study guide the cardiovascular system and disorders ... (ekg): pemasangan

Study Guide Cardiovascular System and Disorders

Department of Medical Education - Faculty of Medicine - Universitas Udayana, 2018

43

LECTURE 9

HYPERTENSIVE URGENCY AND EMERGENCY

dr. AA Ayu Dwi Adelia Yasmin, M.Biomed, SpJP, FIHA

Department of Cardiology and Vascular Medicine

AIMS:

Describe diagnostic approach and management of Hypertensive Urgency and Emergency

LEARNING OUTCOME:

Be able to describe diagnostic approach and management of Hypertensive Urgency and

Emergency.

CURRICULUM CONTENTS:

1. Etiology and pathophysiology of Hypertensive Urgency and Emergency

2. Clinical criteria for Hypertensive Urgency and Emergency

3. Diagnostic approach to Hypertensive Urgency and Emergency

4. Management and prognosis of Hypertensive Urgency and Emergency

ABSTRACT:

Hypertensive emergencies encompass a spectrum of clinical presentations in which

uncontrolled blood pressures lead to progressive or impending end-organ dysfunction. In these

conditions, the blood pressure should be lowered aggressively over minutes to hours.

Neurologic end-organ damage due to uncontrolled blood pressure may include hypertensive

encephalopathy, cerebral vascular accident/ cerebral infarction, subarachnoid hemorrhage,

and/or intracranial hemorrhage. Cardiovascular end-organ damage may include myocardial

ischemia/infarction, acute left ventricular dysfunction, acute pulmonary edema, and/or aortic

dissection. Other organ systems may also be affected by uncontrolled hypertension, which may

lead to acute renal failure/insufficiency, retinopathy, eclampsia, or microangiopathic hemolytic

anemia. The primary goal of the emergency physician is to determine which patients with acute

hypertension are exhibiting symptoms of end-organ damage and require immediate intravenous

parenteral therapy. In contrast, patients presenting with acutely elevated BP (systolic blood

pressure [SBP] >200 mm Hg or diastolic blood pressure [DBP] >120 mm Hg) without symptoms

and whose blood pressure stays significantly elevated to this level on discharge should have

initiation of medical therapy and close follow-up in the outpatient setting. Optimal

pharmacotherapy is dependent upon the specific organ at risk. In patients presenting with

hypertensive emergencies, antihypertensive drug therapy has been shown to be effective in

acutely decreasing blood pressure.

Standard References:

1. Chobanian A, et al. Seventh Report of the Joint National Committee On Prevention,

Detection, Evaluation, And Treatment Of High Blood Pressure. Hypertension.

2003;42:1206–1252.

2. Mancia G, et al. 2013 ESH/ESC Guidelines for the Management of Arterial

Hypertension. European Heart Journal. 2013;34:2159-2219.

Page 44: STUDY GUIDE THE CARDIOVASCULAR SYSTEM AND DISORDERS · study guide the cardiovascular system and disorders ... study guide the cardiovascular system and disorders ... (ekg): pemasangan

Study Guide Cardiovascular System and Disorders

Department of Medical Education - Faculty of Medicine - Universitas Udayana, 2018

44

3. James PA, et al. 2014 Evidence-Based Guideline for the Management of High Blood

Pressure in Adults. Report From the Panel Members Appointed to the Eighth Joint

National Committee (JNC 8). 2014;311(5):507-520.

SCENARIO CASE:

A 67-year-old woman is brought to the emergency room by her son. He states that she has

been “acting strangely” for the last day or so. According to her records, her usual

antihypertensive regimen consists of lisinopril 10 mg PO every day, HCT 25 mg PO every day,

nifedipine extended-release 90 mg PO every day, and atenolol 50 mg every day. Her son

indicates that she ran out of her medication a week ago. On examination, the patient is

confused, somnolent, and complaining of a headache. Her blood pressure is 230/114 bilaterally.

Funduscopic examination shows arteriolar narrowing and indistinct optic disc margins. The lung

examination reveals no rales; the cardiac examination is significant for an S4 gallop and a grade

2 mid-systolic ejection murmur. The abdomen is soft and non-tender, with no bruits. No

peripheral edema is present.

LEARNING TASK:

1. What is the diagnosis of the patient? Please explain the data that support the diagnosis!

2. What is the appropriate management for the patient?

3. How much the blood pressure goal for this patient?

4. Please give the education for patient and family to prevent this condition in the future!

SELF-ASSESSMENT:

1. Please explain the pathophysiology of hypertensive emergency and urgency!

2. What are the signs and symptoms of hypertensive emergency?

3. What are the drug of choice for treatment of hypertensive urgency?

Page 45: STUDY GUIDE THE CARDIOVASCULAR SYSTEM AND DISORDERS · study guide the cardiovascular system and disorders ... study guide the cardiovascular system and disorders ... (ekg): pemasangan

Study Guide Cardiovascular System and Disorders

Department of Medical Education - Faculty of Medicine - Universitas Udayana, 2018

45

LECTURE 10

ACUTE HEART FAILURE

dr. I Kadek Susila Surya Darma, M.Biomed, SpJP, FIHA

Department of Cardiology and Vascular Medicine

AIMS:

Describe to definition, diagnosis, diagnostic test and treatment of Acute Heart Failure.

LEARNING OUTCOME:

1. Can describe definition of Acute Heart Failure

2. Can describe diagnosis of Acute Heart Failure

3. Can describe diagnostic test for Acute Heart Failure

4. Can describe treatment of Acute Heart Failure.

CURRICULUM CONTENT:

Factors triggering Acute Heart Failure

Clinical and diagnostic approach of Acute Heart Failure

Pharmacologic treatment of Acute Heart Failure

Goals of treatment in Acute Heart Failure.

ABSTRACT:

AHF may present as a first occurrence (de novo) or, more frequently, as a consequence of

acute decompensation of chronic HF, and may be caused by primary cardiac dysfunction or

precipitated by extrinsic factors, often in patients with chronic HF. Acute myocardial dysfunction

(ischaemic, inflammatory or toxic), acute valve insufficiency or pericardial tamponade are

among the most frequent acute primary cardiac causes of AHF. Decompensation of chronic HF

can occur without known precipitant factors, but more often with one or more factors, such as

infection, uncontrolled hypertension, rhythm disturbances or non-adherence with drugs/diet.

A large number of overlapping classifications of AHF based on different criteria have been

proposed. The most useful classifications are those based on a clinical presentation at

admission, allowing clinicians to identify patients at high risk of complications, and to manage at

specific targets, which creates a pathway for personalized care in the AHF setting.

The diagnostic workup needs to be started in the pre-hospital setting and continued in the

emergency department (ED) in order to establish the diagnosis in a timely manner and initiate

appropriate management. The greater benefit of early treatment is well established in ACS and

now needs to be considered in the setting of AHF.

In parallel, coexisting life-threatening clinical conditions and/or precipitants that require urgent

treatment/correction need to be immediately identified and managed. Typically, an initial step in

the diagnostic workup of AHF is to rule out alternative causes for the patient’s symptoms and

signs (i.e. pulmonary infection, severe anemia, acute renal failure). AHF is a life-threatening

medical condition, thus rapid transfer to the nearest hospital should be pursued, preferably to a

site with a cardiology department and/or a coronary care/ intensive care unit (CCU/ICU).

Early diagnosis is important in AHF. Therefore, all patients with suspected AHF should have a

diagnostic workup and appropriate pharmacological and non-pharmacological treatment should

Page 46: STUDY GUIDE THE CARDIOVASCULAR SYSTEM AND DISORDERS · study guide the cardiovascular system and disorders ... study guide the cardiovascular system and disorders ... (ekg): pemasangan

Study Guide Cardiovascular System and Disorders

Department of Medical Education - Faculty of Medicine - Universitas Udayana, 2018

46

be started promptly and in parallel. Initial evaluation and continued non-invasive monitoring of

the patient’s vital cardiorespiratory functions, including pulse oximetry, blood pressure,

respiratory rate and a continuous ECG instituted within minutes, is essential to evaluate whether

ventilation, peripheral perfusion, oxygenation, heart rate and blood pressure are adequate.

Urine output should also be monitored, although routine urinary catheterization is not

recommended. Patients with respiratory distress/failure or hemodynamic compromise should be

triaged to a location where immediate respiratory and cardiovascular support can be provided

Standard References:

1. Mann, DL et al. Braunwald’s Heart Disease, 10th ed. Philadelphia, Elsevier Saunders,

2015. p. 429-615

2. Ponikowski et al. 2016 ESC Guidelines For The Diagnosis and Treatment of Acute and

Chronic Heart Failure. European Heart Journal. 2016;ehw128

SELF DIRECTING LEARNING

Basic knowledge that must be known:

1. Factors triggering Acute Heart Failure

2. Clinical and diagnostic approach of Acute Heart Failure

3. Pharmacologic treatment of Acute Heart Failure

4. Goals of treatment in Acute Heart Failure.

SCENARIO: CASE:

A 65-year-old man is hospitalized after he is taken to the emergency department because of

dyspnea and leg edema. He has a longstanding history of hypertension that is treated with

amlodipine. Coronary angiography performed 1 year ago because of chest pain was normal. On

admission, blood pressure is 180/110 mmHg and heart rate is 120/min and regular. Jugular

venous distension is 10 cm while the patient is lying on a stretcher with his head elevated at 45

degrees. He has positive hepatojugular reflex, pitting leg edema, S3 gallop, and rales at basal of

both lungs. No heart murmur is auscultated. Echocardiogram shows left ventricular (LV) ejection

fraction of 20% and LV dilatation. Electrocardiogram shows a left bundle branch block. Serum

electrolytes, hepatic and renal function measurements are normal.

LEARNING TASK :

1. What is the most likely diagnosis of the patient?

2. What is the etiology of the disease?

3. What is the initial treatment for this patient?

SELF ASSESSMENT:

1. Please explain the definition of acute heart failure!

2. What are factors triggering acute heart failure?

3. What is signs and symptoms of heart failure?

4. Please explain clinical profiles of patients with Acute heart failure!

5. Please explain initial treatment for patients with acute heart failure!

6. What is/ are the goal(s) of treatment in acute heart failure?

Page 47: STUDY GUIDE THE CARDIOVASCULAR SYSTEM AND DISORDERS · study guide the cardiovascular system and disorders ... study guide the cardiovascular system and disorders ... (ekg): pemasangan

Study Guide Cardiovascular System and Disorders

Department of Medical Education - Faculty of Medicine - Universitas Udayana, 2018

47

LECTURE 11

CHRONIC HEART FAILURE

dr. I Nyoman Wiryawan, SpJP

Department of Cardiology and Vascular Medicine

AIMS:

Describe to pathophysiology, diagnosis, diagnostic test and treatment Heart Failure.

LEARNING OUTCOME:

1. Can describe pathophysiology of Chronic Heart Failure

2. Can describe diagnosis of Chronic Heart Failure

3. Can describe diagnostic test for Chronic Heart Failure

4. Can describe treatment of Chronic Heart Failure

CURRICULUM CONTENT:

1. Etiology and pathophysiology of Chronic Heart Failure

2. Clinical and diagnostic approach to Chronic Heart Failure

3. Pharmacologic treatment of Chronic Heart Failure

4. Prognosis of Chronic Heart Failure.

ABSTRACT:

Heart failure (HF) can be defined as an abnormality of cardiac structure or function leading to

failure of the heart to deliver oxygen at a rate commensurate with the requirements of the

metabolizing tissues, despite normal filling pressures (or only at the expense of increased filling

pressures). HF is defined, clinically, as a syndrome in which patients have typical symptoms

(e.g. breathlessness, ankle swelling, and fatigue) and signs (e.g. elevated jugular venous

pressure, pulmonary crackles, and displaced apex beat) caused by a structural and/or functional

cardiac abnormality, resulting in a reduced cardiac output and/or elevated intracardiac

pressures at rest or during stress.

The current definition of HF restricts itself to stages at which clinical symptoms are apparent.

Before clinical symptoms become apparent, patients can present with asymptomatic structural

or functional cardiac abnormalities [systolic or diastolic left ventricular (LV) dysfunction], which

are precursors of HF.Recognition of these precursors is important because they are related to

poor outcomes, and starting treatment at the precursor stage may reduce mortality in patients

with asymptomatic systolic LV dysfunction. Demonstration of an underlying cardiac cause is

central to the diagnosis of HF. This is usually a myocardial abnormality causing systolic and/or

diastolic ventricular dysfunction. However, abnormalities of the valves, pericardium,

endocardium, heart rhythm and conduction can also cause HF (and more than one abnormality

is often present). Identification of the underlying cardiac problem is crucial for therapeutic

reasons, as the precise pathology determines the specific treatment used (e.g. valve repair or

replacement for valvular disease, specific pharmacological therapy for HF with reduced EF,

reduction of heart rate in tachy-cardiomyopathy, etc).

The main terminology used to describe HF is historical and is based on measurement of the Left

Ventricle Ejection Fraction (LVEF). HF comprises a wide range of patients, from those with

Page 48: STUDY GUIDE THE CARDIOVASCULAR SYSTEM AND DISORDERS · study guide the cardiovascular system and disorders ... study guide the cardiovascular system and disorders ... (ekg): pemasangan

Study Guide Cardiovascular System and Disorders

Department of Medical Education - Faculty of Medicine - Universitas Udayana, 2018

48

normal LVEF [typically considered as ≥50%; HF with preserved EF (HFpEF)] to those with

reduced LVEF [typically considered as <40%; HF with reduced EF (HFrEF)]. Patients with an

LVEF in the range of 40–49% represent a ‘grey area’, which we now define as HfmrEF (HF mid-

range EF). Differentiation of patients with HF based on LVEF is important due to different

underlying aetiologies, demographics, co-morbidities, and response to therapies.

The diagnosis of HFpEF is more challenging than the diagnosis of HFrEF. Patients with HFpEF

generally do not have a dilated LV, but instead, often have an increase in LV wall thickness

and/or increased left atrial (LA) size as a sign of increased filling pressures. Most have

additional ‘evidence’ of impaired LV filling or suction capacity, also classified as diastolic

dysfunction, which is generally accepted as the likely cause of HF in these patients (hence the

term ‘diastolic HF’). However, most patients with HFrEF (previously referred to as ‘systolic HF’)

also have diastolic dysfunction, and subtle abnormalities of systolic function have been shown in

patients with HFpEF. Hence the preference for stating preserved or reduced LVEF over

preserved or reduced ‘systolic function’.

Based on ESC guidelines for the diagnosis and treatment of acute and chronic HF 2016, the

term HF is used to describe the symptomatic syndrome, graded according to the New York

Heart Association (NYHA) functional classification, although a patient can be rendered

asymptomatic by treatment. In these guidelines, a patient who has never exhibited the typical

symptoms and/or signs of HF and with a reduced LVEF is described as having asymptomatic

LV systolic dysfunction. Patients who have had HF for some time are often said to have ‘chronic

HF’. A treated patient with symptoms and signs that have remained generally unchanged for at

least 1 month is said to be ‘stable’. If chronic stable HF deteriorates, the patient may be

described as ‘decompensated’ and this may happen suddenly or slowly, often leading to

hospital admission, an event of considerable prognostic importance. New-onset (‘de novo’) HF

may also present acutely, for example, as a consequence of acute myocardial infarction (AMI),

or in a subacute (gradual) fashion, for example, in patients with a dilated cardiomyopathy

(DCM), who often have symptoms for weeks or months before the diagnosis becomes clear.

Although symptoms and signs of HF may resolve, the underlying cardiac dysfunction may not,

and patients remain at the risk of recurrent ‘decompensation’. Congestive HF’ is a term that is

sometimes used and may describe acute or chronic HF with evidence of volume overload. Many

or all of these terms may be accurately applied to the same patient at different times, depending

upon their stage of illness.

The prevalence of HF depends on the definition applied, but is approximately 1–2% of the adult

population in developed countries has HF, with the prevalence rising to ≥10% among persons

70 years of age or older. Among people >65 years of age presenting to primary care with

breathlessness on exertion, one in six will have unrecognized HF (mainly HFpEF). The lifetime

risk of HF at age 55 years is 33% for men and 28% for women. The proportion of patients with

HFpEF ranges from 22 to 73%, depending on the definition applied the clinical setting (primary

care, hospital clinic, hospital admission), age and sex of the studied population, previous

myocardial infarction. There are many causes of HF, and these vary in different parts of the

world. At least half of patients with HF have a low left ventricular ejection fraction (LVEF).

Coronary artery disease (CAD) is the cause of approximately two-thirds of cases of systolic HF,

although hypertension and diabetes are probably contributing factors in many cases. There are

many other causes of systolic HF, which include previous viral infection (recognized or

unrecognized), alcohol abuse, chemotherapy (e.g. doxorubicin or trastuzumab), and ‘idiopathic’

Page 49: STUDY GUIDE THE CARDIOVASCULAR SYSTEM AND DISORDERS · study guide the cardiovascular system and disorders ... study guide the cardiovascular system and disorders ... (ekg): pemasangan

Study Guide Cardiovascular System and Disorders

Department of Medical Education - Faculty of Medicine - Universitas Udayana, 2018

49

dilated cardiomyopathy (although the cause is thought to be unknown, some of these cases

may have a genetic basis).

Data on temporal trends based on hospitalized patients suggest that the incidence of HF may

be decreasing, more for HFrEF than for HFpEF. HFpEF and HFrEF seem to have different

epidemiological and etiological profiles. Compared with HFrEF, patients with HFpEF are older,

more often women and more commonly have a history of hypertension and atrial fibrillation

(AF), while a history of myocardial infarction is less common.

Two key neurohumoral systems activated in HF are the renin–angiotensin–aldosterone system

and sympathetic nervous system. In addition to causing further myocardial injury, these

systemic responses have detrimental effects on the blood vessels, kidneys, muscles, bone

marrow, lungs, and liver, and create a pathophysiological ‘vicious cycle’, accounting for many of

the clinical features of the HF syndrome, including myocardial electrical instability. Interruption

of these two key processes is the basis of much of the effective treatment of HF.

The diagnosis of HF can be difficult, especially in the early stages. Although symptoms bring

patients to medical attention, many of the symptoms of HF are non-specific and do not,

therefore help discriminate between HF and other problems. Symptoms that are more specific

(i.e. orthopnea and paroxysmal nocturnal dyspnea) are less common, especially in patients with

milder symptoms, and are, therefore, insensitive. Many of the signs of HF result from sodium

and water retention, and are, therefore, also not specific. Peripheral edema has other causes as

well and is particularly non-specific. Signs resulting from sodium and water retention (e.g.

peripheral edema) resolve quickly with diuretic therapy (i.e. may be absent in patients receiving

such treatment, making it more difficult to assess patients already treated in this way). More

specific signs, such as elevated jugular venous pressure and displacement of the apical

impulse, are harder to detect and, therefore, less reproducible (i.e. agreement between different

doctors examining the same patient may be poor).

Symptoms and signs may be particularly difficult to identify and interpret in obese individuals, in

the elderly, and in patients with chronic lung disease. The patient’s medical history is also

important. HF is unusual in an individual with no relevant medical history (e.g. a potential cause

of cardiac damage), whereas certain features, particularly previous myocardial infarction, greatly

increase the likelihood of HF in a patient with appropriate symptoms and signs. These points

highlight the need to obtain objective evidence of a structural or functional cardiac abnormality

that is thought to account for the patient’s symptoms and signs, to secure the diagnosis of HF.

Once the diagnosis of HF has been made, it is important to establish the cause, particularly

specific correctable causes. Symptoms and signs are important in monitoring a patient’s

response to treatment and stability over time. Persistence of symptoms despite treatment

usually indicates the need for additional therapy, and worsening of symptoms is a serious

development (placing the patient at risk of urgent hospital admission and death) and merits

prompt medical attention.

The echocardiography and electrocardiography (ECG) are the most useful tests in patients with

suspected HF. The echocardiogram provides immediate information on chamber volumes,

ventricular systolic and diastolic function, wall thickness, and valve function. This information is

crucial in determining appropriate treatment (e.g. an ACE inhibitor and beta-blocker for systolic

dysfunction or surgery for aortic stenosis (Valvular Heart Disease)). The ECG shows the heart

rhythm and electrical conduction, i.e. whether there is sinoatrial disease, atrioventricular (AV)

block or abnormal intraventricular conduction. These findings are also important for decisions

Page 50: STUDY GUIDE THE CARDIOVASCULAR SYSTEM AND DISORDERS · study guide the cardiovascular system and disorders ... study guide the cardiovascular system and disorders ... (ekg): pemasangan

Study Guide Cardiovascular System and Disorders

Department of Medical Education - Faculty of Medicine - Universitas Udayana, 2018

50

about treatment (e.g. rate control and anticoagulation for AF, pacing for bradycardia, or CRT if

the patient has LBBB). The ECG may also show evidence of LV hypertrophy or Q waves

(indicating loss of viable myocardium), giving a possible clue to the etiology of HF. HF is very

unlikely (likelihood, 2%) in patients presenting acutely and with a completely normal ECG. The

information provided by these two tests will permit an initial working diagnosis and treatment

plan in the majority of patients. Routine biochemical and hematological investigations are also

important, partly to determine whether renin-angiotensin– aldosterone blockade can be initiated

safely (renal function and potassium) and to exclude anemia (which can mimic or aggravate HF)

and because they provide other useful information. Because the signs and symptoms of HF are

so non-specific, many patients with suspected HF referred for echocardiography are not found

to have an important cardiac abnormality. Where the availability of echocardiography is limited,

an alternative approach to diagnosis is to measure the blood concentration of a natriuretic

peptide, a family of hormones secreted in increased amounts when the heart is diseased

(stretched) or the load on any chamber is increased (e.g. by AF, pulmonary embolism, and

some non-cardiovascular conditions, including renal failure). A chest X-ray is of limited use in

the diagnostic workup of patients with suspected HF. It is probably most useful in identifying an

alternative pulmonary explanation for a patient’s symptoms and signs. It may, however, show

pulmonary venous congestion or edema in a patient with HF. It is important to note that

significant LV systolic dysfunction may be present without cardiomegaly on the chest X-ray.

Estimation of prognosis for morbidity, disability, and death helps patients, their families and

clinicians decide on the appropriate type and timing of therapies (in particular, decisions about a

rapid transition to advanced therapies) and assists with the planning of health and social

services and resources. Many variables provide prognostic information, although most of this

can be obtained from readily available data such as age, etiology, NYHA class, LVEF, key co-

morbidities (renal dysfunction, diabetes mellitus, anemia, hyperuricemia), and plasma natriuretic

peptide concentration. Clearly, these variables change over time, as does prognosis so that

precise risk stratification in HF remains challenging. Assessment of prognosis is particularly

important when counseling patients about devices and surgery (including transplantation) and in

planning end-of-life care with patients, their family, and caregivers.

The goals of treatment in patients with established HF are to relieve symptoms and signs (e.g.

edema), prevent hospital admission, and improve survival and finally improved quality of life.

Although the focus of clinical trials was previously mortality, it is now recognized that preventing

HF hospitalization is important for patients and healthcare systems. Reduction in mortality and

hospital admission rates both reflect the ability of effective treatments to slow or prevent

progressive worsening of HF. This is often accompanied by reverse LV remodeling and a

reduction in circulating natriuretic peptide concentrations. The relief of symptoms, improvement

in the quality of life, and increase in functional capacity are also of the utmost importance to

patients, but they have not been the primary outcome in most trials. Three neurohumoral

antagonists— an ACE inhibitor [or angiotensin receptor blocker (ARB)], a beta-blocker, and a

mineralocorticoid receptor antagonist (MRA)—are fundamentally important in modifying the

course of systolic HF and should at least be considered in every patient. They are commonly

used in conjunction with a diuretic given to relieve the symptoms and signs of congestion.

Page 51: STUDY GUIDE THE CARDIOVASCULAR SYSTEM AND DISORDERS · study guide the cardiovascular system and disorders ... study guide the cardiovascular system and disorders ... (ekg): pemasangan

Study Guide Cardiovascular System and Disorders

Department of Medical Education - Faculty of Medicine - Universitas Udayana, 2018

51

Standard References:

1. Libby P, Bonow RO, Mann DL, Zipes DP eds. Heart Failure. In: Braunwald’s Heart

Disease, 10th ed. Philadelphia, Elsevier Saunders, 2015. p. 429-615

2. Ponikowski et al. ESC Guidelines for The Diagnosis and Treatment of Acute and

Chronic Heart Failure. European Journal Heart Failure. 2016 August;18(8): 891-975.

3. Miranda D, Lewis GD, Fifer MA. Heart Failure. In: Lilly LS ed. Pathophysiology of Heart

Disease. 6th ed. Philadelphia: Wolters Kluwer; 2016:220-246

SELF DIRECTING LEARNING

Basic knowledge that must be known:

1. Etiology of Chronic Heart Failure

2. Pathophysiology of Chronic Heart Failure

3. Clinical and diagnostic approach to Chronic Heart Failure

4. Pharmacologic treatment of Chronic Heart Failure

5. Prognosis of Chronic Heart Failure

SCENARIO: CASE:

A 65 years old man is hospitalized after he is taken to the emergency department because of

dyspnea and leg edema. He has a longstanding history of hypertension that is treated with

amlodipine. Coronary angiography performed 1 year ago because of chest pain was normal. On

admission, blood pressure was 180/110 mmHg and heart rate was 120/min regularly. Jugular

venous distension is 10 cm while the patient is lying on a stretcher with his head elevated at 45

degrees. He has positive hepatojugular reflex, pitting leg edema, S3 gallop, and rales at basal of

both lungs. No heart murmurs are auscultated. Echocardiography examination showed left

ventricular (LV) ejection fraction of 20% and LV dilatation. Electrocardiography result was a left

bundle branch block. Serum electrolytes, hepatic and renal function measurements are normal.

LEARNING TASK:

1. What is the most likely diagnosis of the patient?

2. What is the etiology of the disease?

3. What is the pharmacological treatment for this patient?

SELF ASSESSMENT:

1. Please explain the definition of heart failure!

2. Please explain terminology heart failure related to left ventricular ejection fraction!

3. Please explain terminology heart failure related to the symptomatic severity of heart

failure!

4. What are signs and symptoms of heart failure?

5. Please explain general diagnostic test in patients with suspected heart failure!

6. What are can impact the prognosis of chronic heart failure?

7. Please explain pharmacological treatment for patients with chronic heart failure!

Page 52: STUDY GUIDE THE CARDIOVASCULAR SYSTEM AND DISORDERS · study guide the cardiovascular system and disorders ... study guide the cardiovascular system and disorders ... (ekg): pemasangan

Study Guide Cardiovascular System and Disorders

Department of Medical Education - Faculty of Medicine - Universitas Udayana, 2018

52

LECTURE 12 INTRODUCTION TO ARRHYTHMIA

dr. I Made Putra Swi Antara, SpJP(K), FIHA

Department of Cardiology and Vascular Medicine

AIMS:

Able to understand and diagnose arrhythmias.

LEARNING OUTCOME:

Can describe and diagnose arrhythmias.

CURRICULUM CONTENTS:

1. Etiology and pathophysiology of Arrhythmias.

2. Clinical approach to Arrhythmias.

3. Treatment of Arrhythmias.

4. Prognosis of Arrhythmias.

ABSTRACT:

An arrhythmia (or dysrhythmia) is a disturbance of the electrical rhythm of the heart. Most

arrhythmias are benign and are only troublesome because of the symptoms they cause.

However, some arrhythmias are dangerous and require treatment to prevent hemodynamic

compromise or cardiac arrest, and it is important to recognize these.

In the management of clinical arrhythmias, the physician must evaluate and treat the whole

patient, not just the rhythm disturbance. Some arrhythmias are hazardous to the patient,

regardless of the clinical setting (e.g., ventricular fibrillation/VF), whereas others are hazardous

because of the clinical setting (e.g., rapidly conducted atrial fibrillation in a patient with severe

coronary artery stenosis). Patients with cardiac rhythm disturbances can present with various

complaints, but symptoms such as palpitations, syncope, presyncope, or congestive heart

failure commonly cause them to seek a physician’s help. Their awareness of palpitations and of

a regular or irregular cardiac rhythm varies greatly. A careful drug and dietary history should

also be sought; some nasal decongestants can provoke tachycardia episodes, whereas beta-

adrenergic blocking eye drops for the treatment of glaucoma can drain into tear ducts, be

absorbed systemically, and precipitate syncope caused by bradycardia.

Examination of the patient during a symptomatic episode can be revealing. Clearly, heart rate

and blood pressure are key measurements to make. Assessment of the jugular venous

pressure and waveform can disclose the rapid oscillations of atrial flutter or “cannon” A waves

indicative of contraction of the right atrium against a closed tricuspid valve in patients with AV

dissociation in disorders such as complete heart block or Ventricular Tachycardia/VT. Variations

in the intensity of the first heart sound and systolic blood pressure have the same implications.

The electrocardiogram/ ECG is the primary tool in arrhythmia analysis; an Electrophysiology

Study/EPS, in which intracardiac catheters are used to record activity from several regions of

the heart at one time, is more definitive. Initially, a 12-lead ECG is recorded. In addition, a long

continuous recording with use of the lead that shows distinct P waves is often helpful for closer

analysis; most commonly, this is one of the inferior leads (2, 3, aVF), V1, or aVR. The ECG

obtained during an episode of arrhythmia may be diagnostic by itself, obviating the need for

Page 53: STUDY GUIDE THE CARDIOVASCULAR SYSTEM AND DISORDERS · study guide the cardiovascular system and disorders ... study guide the cardiovascular system and disorders ... (ekg): pemasangan

Study Guide Cardiovascular System and Disorders

Department of Medical Education - Faculty of Medicine - Universitas Udayana, 2018

53

further diagnostic testing the following additional tests can be used to evaluate patients who

have cardiac arrhythmias. The physician’s choice of which test to use depends on the clinical

circumstances. For example, a patient with multiple daily episodes of presyncope is likely to

have an event recorded on a 24-hour ambulatory electrocardiographic (Holter) monitor, whereas

in a patient who complains of infrequent anxiety- or exercise-induced palpitations, an

exercise/treadmill stress testing may be more likely to provide a diagnosis.

Normal sinus rhythm is arbitrarily limited to impulse formation beginning in the sinus node at

rates between 60 and 100 beats/min. Infants and children generally have faster heart rates than

adults do, both at rest and during exercise. Rates below 50 beats/min are considered to be

bradycardia, and rates above 100 beats/min are considered to be tachycardia.

During sinus tachycardia, the sinus node exhibits a discharge frequency between 100 and 180

beats/min, but it can be higher with extreme exertion and in young individuals. The maximum

heart rate achieved during strenuous physical activity decreases with age from about 200

beats/min at 20 years to less than 140 beats/min at 80 years.

Tachyarrhythmias are broadly characterized as supraventricular tachycardia (SVT), defined as a

tachycardia in which the driving circuit or focus originates, at least in part, in tissue above the

level of the ventricle (i.e., sinus node, atria, AV node, or His bundle), and ventricular tachycardia

(VT), defined as a tachycardia in which the driving circuit or focus solely originates in ventricular

tissue or Purkinje fibers. Because of differences in prognosis and management, it is critical to

making the distinction between SVT and VT in the acute management of a tachyarrhythmia. In

general (with the exception of idiopathic VT), VT often carries a much graver prognosis, usually

implies the presence of significant heart disease, results in more profound hemodynamic

compromise, and therefore requires immediate attention and measures to revert to sinus

rhythm. On the other hand, SVT is usually not lethal and often does not result in hemodynamic

collapse; therefore, more conservative measures can be applied initially to convert to sinus

rhythm. Supraventricular tachycardia (SVTs) are almost always benign. Initial management of

SVT comprises of the Valsalva maneuver, carotid sinus pressure or administration of

intravenous adenosine. Beta-blocker and verapamil reduce symptoms significantly in two-thirds

of patients with recurrent SVT. Radio-frequency ablation/RFA should be considered for all

patients with frequent SVT.

Atrial fibrillation (AF) is a supraventricular arrhythmia characterized by low-amplitude baseline

oscillations (fibrillatory or F waves) and an irregularly irregular ventricular rhythm on an ECG. AF

is the most common arrhythmia treated in clinical practice and the most common arrhythmia for

which patients are hospitalized; approximately 33% of arrhythmia-related hospitalizations are for

AF. The symptoms of AF vary widely between patients, ranging from none to severe and

functionally disabling symptoms. The most common symptoms of AF are palpitations, fatigue,

dyspnea, effort intolerance, and lightheadedness.

Atrial flutter is less common than atrial fibrillation. The atrial rate during typical atrial flutter is

usually 250 to 350 beats/min, although it is occasionally slower, particularly when the patient is

treated with antiarrhythmic drugs, which can reduce the rate to about 200 beats/min. In typical

atrial flutter, the ECG reveals identically recurring, regular, sawtooth flutter waves and evidence

of continual electrical activity (lack of an isoelectric interval between flutter waves), often best

visualized in leads II, III, aVF, or V1.

Management of atrial fibrillation and flutter is generally rate-control strategy. It is directed at

limiting the ventricular response to atrial fibrillation by using AV node blocking drugs, such as

Page 54: STUDY GUIDE THE CARDIOVASCULAR SYSTEM AND DISORDERS · study guide the cardiovascular system and disorders ... study guide the cardiovascular system and disorders ... (ekg): pemasangan

Study Guide Cardiovascular System and Disorders

Department of Medical Education - Faculty of Medicine - Universitas Udayana, 2018

54

digoxin, beta-blockers, and verapamil. Cardioversion and anti-arrhythmic drugs are used to

restore and maintain sinus rhythm. Anti-coagulation with warfarin should be considered for

patients with atrial fibrillation and risk factors for thromboembolic stroke by using a scoring

system (CHA2DS2-VASc).

Premature complexes are among the most common causes of an irregular pulse. They can

originate from any area in the heart—most frequently from the ventricles, less often from the

atria and the AV junctional area, and rarely from the sinus node. Although premature complexes

arise commonly in normal hearts, they are more often associated with structural heart disease

and increase in frequency with age. The diagnosis of premature atrial complexes (PACs) is

made on the ECG by the presence of a premature P wave with a PR interval of 120

milliseconds (except in Wolff- Parkinson-White syndrome, in which case the PR interval is

usually shorter than 120 milliseconds). Although the contour of a premature P wave can

resemble that of a normal sinus P wave, it generally differs.

The prevalence of premature ventricular complexes increases with age; they are associated

with male gender and a reduced serum potassium concentration. Symptoms of palpitations or

discomfort in the neck or chest can result because of the greater than normal contractile force of

the post-extrasystolic beat or the feeling that the heart has stopped during the long pause after

the premature complex. A PVC is characterized by the premature occurrence of a QRS complex

that is abnormal in shape and has a duration usually exceeding the dominant QRS complex,

generally longer than 120 milliseconds. In most patients, PVCs (occurring as single PVCs,

bigeminy, or trigeminy but excluding non-sustained VT) do not need to be treated and treatment

is usually dictated by the presence of symptoms attributable to the PVCs.

In general, the specific type, prognosis, and management of ventricular tachycardia (VT)

depend on whether underlying structural heart disease is present. The electrocardiographic

diagnosis of VT is suggested by the occurrence of a series of three or more consecutive,

abnormally shaped PVCs whose duration exceeds 120 milliseconds, with the ST-T vector

pointing opposite the major QRS deflection. Symptoms occurring during VT depend on the

ventricular rate, duration of tachycardia, and presence and extent of the underlying heart

disease and peripheral vascular disease. VT can occur in several forms: short, asymptomatic,

nonsustained episodes; sustained, hemodynamically stable events, generally occurring at

slower rates or in otherwise normal hearts; or unstable runs, often degenerating into VF. The

dramatic changes in the management of VT and aborted sudden death during the past decade

have been fueled by several large clinical trials and development of the Implantable

Cardioverter Defibrillator/ICD. Management decisions can be stratified into those involved in

acute management (or termination) and those involved in long-term therapy (or prevention of

recurrence or sudden death).

Ventricular fibrillation (VF) occurs in various clinical situations but most commonly in association

with coronary artery disease and as a terminal event. Ventricular flutter or VF results in

faintness, followed by loss of consciousness, seizures, apnea, and eventually, if the rhythm

continues untreated, death. The blood pressure is unobtainable, and heart sounds are usually

absent. These arrhythmias represent severe derangements of the heartbeat that usually

terminate fatally within 3 to 5 minutes unless corrective measures are undertaken promptly.

Ventricular flutter is manifested as a sine wave in appearance—regular large oscillations

occurring at a rate of 150 to 300 beats/min (usually about 200). The distinction between rapid

VT and ventricular flutter can be difficult and is usually of academic interest only. Hemodynamic

Page 55: STUDY GUIDE THE CARDIOVASCULAR SYSTEM AND DISORDERS · study guide the cardiovascular system and disorders ... study guide the cardiovascular system and disorders ... (ekg): pemasangan

Study Guide Cardiovascular System and Disorders

Department of Medical Education - Faculty of Medicine - Universitas Udayana, 2018

55

collapse is present with both. VF is recognized by the presence of irregular undulations of

varying contour and amplitude. Distinct QRS complexes, ST segments, and T waves are

absent. Fine-amplitude fibrillatory waves (0.2 mV) are present with prolonged VF. These fine

waves identify patients with worse survival rates and are sometimes confused with asystole.

Management should follow basic life support and advanced cardiac life support guidelines.

Bradyarrhythmia is arbitrarily defined as a heart rate below 60 beats/min. In some cases,

bradyarrhythmia is physiologic, as in well-conditioned athletes with low resting heart rates or

type I AV block during sleep, and in other cases are pathologic. Like tachyarrhythmias,

bradyarrhythmia can be categorized on the basis of the level of disturbance in the hierarchy of

the normal impulse generation and conduction system (from the sinus node to AV node to His-

Purkinje system). Sinus bradycardia exists in an adult when the sinus node discharges at a rate

slower than 60 beats/min. P waves have a normal contour and occur before each QRS

complex, usually with a constant PR interval longer than 120 milliseconds. Sinus arrhythmia

often coexists.

Heart block is a disturbance of impulse conduction that can be permanent or transient,

depending on the anatomic or functional impairment. It must be distinguished from interference,

a normal phenomenon that is a disturbance of impulse conduction caused by physiologic

refractoriness resulting from in excitability caused by a preceding impulse. Interference or block

can occur at any site where impulses are conducted, but they are recognized most commonly

between the sinus node and atrium (SA block), between the atria and ventricles (AV block),

within the atria (intra-atrial block), or within the ventricles (intraventricular block). An AV block

exists when the atrial impulse is conducted with the delay or is not conducted at all to the

ventricle when the AV junction is not physiologically refractory. In some cases of bundle branch

block, the impulse may only be delayed and not completely blocked in the bundle branch, yet

the resulting QRS complex may be indistinguishable from a QRS complex generated by a

complete bundle branch block.

Main Reference:

Mann, DL et all. Braunwald’s Heart Disease, 10th ed. Philadelphia, Elsevier Saunders, 2015. p.

687-837

Mandatory Reading:

Evans, JDW et all. Cardiac Electrophysiology and Arrhythmia. In: Crash Course in

Cardiovascular System 4th Edition. 2012. P 27-42

Available at: http://goo.gl/DY79rx

SELF DIRECTED LEARNING

Basic knowledge that must be known:

1. Understanding of Cardiac Electrophysiologic Properties

2. Basic Understanding of ECGs

3. Definition and Mechanisms of Arrhythmias

4. Clinical approach for Arrhythmias

5. Pharmacological and Procedural Treatments for Arrhythmias

Page 56: STUDY GUIDE THE CARDIOVASCULAR SYSTEM AND DISORDERS · study guide the cardiovascular system and disorders ... study guide the cardiovascular system and disorders ... (ekg): pemasangan

Study Guide Cardiovascular System and Disorders

Department of Medical Education - Faculty of Medicine - Universitas Udayana, 2018

56

SCENARIO FOR DISCUSSION

CASE 1:

A 45-year old gentleman presented with an irregular heartbeat and dizzy. On physical

examination, the blood pressure was 115/75 mmHg; heart rate was 148 beats-per-minute,

irregular and pulse rate was 102 beats-per-minute, irregular. S1 and S2 were single, grade 3/6

rumbling diastolic murmur was heard at apex cordis.

LEARNING TASK:

1. What is the most likely arrhythmia found in this patient?

2. What is the terminology of differentiation between irregular higher heart rate and

irregular lower pulse rate?

CASE 2:

A 32-year old gentleman was brought to the emergency department after an out-of-hospital

cardiac arrest (OOHCA) at 4 am on his bed and was successfully resuscitated. No remarkable

health history or precipitating factors were present, nor any risk factors for CAD. Initial rhythm

observed by the emergency response team was VF and was defibrillated twice to convert back

to sinus rhythm. Physical exam was normal. Recorded ECG doesn’t show any apparent

arrhythmia or abnormality.

1. What are the differential cardiac diagnoses for this patient?

2. What abnormality is looked for in the ECG to support the diagnosis?

3. What treatment options are needed for this patient to prevent recurrence of sudden

cardiac death?

Page 57: STUDY GUIDE THE CARDIOVASCULAR SYSTEM AND DISORDERS · study guide the cardiovascular system and disorders ... study guide the cardiovascular system and disorders ... (ekg): pemasangan

Study Guide Cardiovascular System and Disorders

Department of Medical Education - Faculty of Medicine - Universitas Udayana, 2018

57

LECTURE 13a COMMON PERIPHERAL VASCULAR AND LYMPHATIC DISEASE

Dr. dr. Semadi, Sp.B, Sp.BTKV

Department of Surgeon

AIMS:

Describe the basic principles of surgery in cardiac diseases

LEARNING OUTCOME:

1. Can describe the basic principles of cardiac surgery

2. Can describe the basic aspect of cardiac surgery

3. Can describe the cardiac diseases who need surgery

CURRICULUM CONTENTS:

1. Surgery of the congenital heart

2. Surgery of the acquired heart diseases

ABSTRACT:

Atrial septal defects (ASD) and ventricular septal defect (VSD) and others are the congenital

cardiac anomaly. The intracardiac defects make the shunt and blood flow through the shunt

from right-to-left or reverse of the heart.

Atrial septal defects (ASD) are most common in the vicinity of the fossa ovalis. Septum

secundum defects, the typical patent foramen ovale, account for 10-15% of all cardiac

anomalies. Normal left atrial pressure is slightly greater than right atrial pressure, a left-to-right

shunt occurs through an open ASD, oxygenated blood from the left side of the heart is shunted

to the right side, thus not associated with cyanosis. An ASD is usually compatible with normal

life, except at an extreme exercise, cardiac disease, or pulmonary disease alter chamber

pressures, a right-to-left shunt will produce cyanosis.

Ventricular septal defect (VSD) usually happens at the upper membranous portion that

composed of connective tissue continuous with the annulus fibrosus. A small VSD may result in

an inconsequential left-to-right shunt.

In the presence of pulmonary stenosis, a VSD produces a right-to-left shunt with cyanosis and

the blue-baby syndrome. A large VSD is a principal factor in Tetralogy of Fallot.

Patent ductus arteriosus (PDA) is a persistence of the fetal connection (ductus arteriosus)

between the aorta and pulmonary artery after birth, resulting in a left-to-right shunt. Symptoms

may include failure to thrive, poor feeding, tachycardia and tachypnea due to a lung infection.

Others cardiac diseases are more common as coronary arterial diseases (CAD), Acquired Valve

diseases of rheumatic heart disease and congenital valve anomaly

Standard References:

Stuart W. Jamieson and Norman E Shumway: Cardiac Surgery in Rob & Smith’s Operative

Surgery, 4th edition. Butterworths London, 2004.

SELF DIRECTING LEARNING

Basic knowledge that must be known is cardiac surgery in principles

Page 58: STUDY GUIDE THE CARDIOVASCULAR SYSTEM AND DISORDERS · study guide the cardiovascular system and disorders ... study guide the cardiovascular system and disorders ... (ekg): pemasangan

Study Guide Cardiovascular System and Disorders

Department of Medical Education - Faculty of Medicine - Universitas Udayana, 2018

58

SCENARIO:

CASE 1;

This baby aged 4 months has been known to have a cardiac murmur since birth. He was born 8

weeks prematurely and developed respiratory distress requiring high oxygen concentration for

the first week. Since then he has feed satisfactorily but height and weight growth have been

poor even allowing for prematurity. The diagnosis after examination and investigations: Patent

Ductus Arteriosus (PDA).

LEARNING TASK I

1. How to prepare if the patient has surgery?

2. What is cardiac surgery category for PDA closure?

3. PDA commonly concomitant with a congenital anomaly. Is it every PDA have surgery to

close the shunt?

4. After an operation to close the PDA, why is there a risk of the patient becoming hoarse?

CASE 2 :

This 13-year-old girl was recently found to have a cardiac murmur. She has been generally

healthy with good growth, but on questioning her mother admitted she has noticed that girl

tends to tire easily with exercise. The diagnosis of examination and investigations: Atrial Septal

Defect (ASD)

LEARNING TASK II:

1. What is cardiac surgery category for ASD closure?

2. What the difference between close and open cardiac surgery?

3. After ASD was closed, why the patient getting good growth of the body?

4. And why is the patient after ASD closure getting arrhythmia?

CASE 3 :

A 2-year-old boy was admitted to the hospital for evaluation of a heart murmur previously detect

at birth. He was less active than other children his age, but although over-exertion was followed

frequently by cyanosis of the lips and nails, there was no history of unconsciousness. Initial

examination revealed a thin, physically retarded, cyanotic child with no respiratory difficulty.

There was moderate clubbing of the fingers. A harsh systolic murmur was maximal over the

mid-precordial area. The first heart sound was normal while the second was single, distinct and

loud.The lungs were clear. X-ray showed a normal sized heart dominated by a boot-shaped

right ventricular outflow tract. Diagnosis of Tetralogy of Fallot.

LEARNING TASK III:

1. The cardiac anomaly is PS, VSD, Overriding aorta and RVH. How do repair it?

2. Why is the patient becoming worse after the surgical repair of the defect?

SELF-ASSESSMENT:

1. Describe the principle cardiac surgery!

2. Describe the principle coronary heart surgery!

3. Describe the principle of Valve surgery!

Page 59: STUDY GUIDE THE CARDIOVASCULAR SYSTEM AND DISORDERS · study guide the cardiovascular system and disorders ... study guide the cardiovascular system and disorders ... (ekg): pemasangan

Study Guide Cardiovascular System and Disorders

Department of Medical Education - Faculty of Medicine - Universitas Udayana, 2018

59

LECTURE 13b TREATMENT OF AORTA AND ARTERIAL DISEASES

Dr. dr. Semadi, Sp.B, Sp.BTKV

Department of Surgeon

AIMS:

Describe the basic principles of surgery in aorta and arteries.

LEARNING OUTCOME:

1. Can describe the basic principles of aorta and arteries.

2. Can describe the basic aspect of aorta arteries.

3. Can describe the aorta and arteries diseases who need surgery.

CURRICULUM CONTENTS:

1. Surgery of the aneurysm of the aorta.

2. Surgery of the peripheral artery diseases.

ABSTRACT:

Atherosclerosis is the usual cause of vascular diseases. The aneurysm of the aorta is dilated of

aorta lumen over one and a half size of the normal lumen of the aorta. The aorta can

enlargement, elongated and tortuous with or without thrombus in the lumen of the aorta. It can

be found in thoracic region or abdominal region or both. The patient got the pain in the chest or

abdominal pain depend on the aneurysm position. If you found the abdominal aortic aneurysm

(triple A), the large pulsatile tumor was found on central topographic of the abdomen. The

patient becomes dangerous if an aortic aneurysm ruptured and the patient getting haemorrhagic

shock.

Atherosclerosis can cause the peripheral artery diseases. The artery becomes aneurysm,

stenosis and occluded. If the artery got occlusion on middle size of that, the distal part of the

organ will ischemic and become the death of the tissue that calls gangrene.

Standard References:

Allan D. Callow, Calvin B. Erust.: Vascular surgery, Theory, and Practice, Prentice- Hall

International Inc.London, 1995

SELF DIRECTING LEARNING

Basic knowledge that must be known is vascular surgery in principles.

SCENARIO: CASE 1;

Old man, he was a pain on the abdomen and the tumor was found on abdomen palpation. The

tumor was pulsatile and 7 cm in diameter and fixed. The blood pressure of the patient got high.

The diagnose of the disease is triple A with stable hemodynamically

LEARNING TASK I:

a) How to diagnose the patient?

b) How to prepare if the patient has surgery?

Page 60: STUDY GUIDE THE CARDIOVASCULAR SYSTEM AND DISORDERS · study guide the cardiovascular system and disorders ... study guide the cardiovascular system and disorders ... (ekg): pemasangan

Study Guide Cardiovascular System and Disorders

Department of Medical Education - Faculty of Medicine - Universitas Udayana, 2018

60

c) How to do to enlargement of the aorta?

d) What are the complications of aorta surgery?

CASE 2 :

This 43-year-old man was recently found to have a cold of feet. He has been generally healthy

with pain on both legs if he walking for while he was heavy smoking from teenager. The

diagnosis after physical examination that concludes: peripheral artery diseases of both popliteal

artery

LEARNING TASK II:

1. What will you do to investigate the patient for definitive diagnosis?

2. What will you do to improve the blood flow to the distal end of feet?

3. How the prognosis and recurrent rate?

SELF-ASSESSMENT:

1. Describe the principle of vascular surgery!

2. Describe the principle arterial repair!

3. Describe the principle of care after vascular surgery!

Page 61: STUDY GUIDE THE CARDIOVASCULAR SYSTEM AND DISORDERS · study guide the cardiovascular system and disorders ... study guide the cardiovascular system and disorders ... (ekg): pemasangan

Study Guide Cardiovascular System and Disorders

Department of Medical Education - Faculty of Medicine - Universitas Udayana, 2018

61

LECTURE 14 VALVULAR HEART DISEASE

dr. I B Rangga Wibhuti, M.Biomed, SpJP, FIHA, FASE

Department of Cardiology and Vascular Medicine

AIMS:

Able to diagnose and manage Valvular Heart Disease (VHD).

LEARNING OUTCOME:

1. Able to combine history taking, physical examination, and supportive examinations to

make initial/working diagnosis of VHD patients, and able to suggest further

examination(s) needed to diagnose VHD.

2. Able to manage VHD patients in short term and long-term management.

3. Able to combine history taking, physical examination, and supportive examinations to

diagnose the patients.

CURRICULUM CONTENT:

1. Etiology, pathophysiology, and clinical spectrum of VHD.

2. Indication and interpretation of supportive examination(s)for VHD.

3. Management of VHD patients.

ABSTRACT:

Valvular heart disease is characterized by damage to or a defect in one of the four heart valves:

the mitral, aortic, tricuspid or pulmonary. The mitral and tricuspid valves control the flow of

blood between the atria and the ventricles (the upper and lower chambers of the heart). The

pulmonary valve controls the flow of blood from the heart to the lungs, and the aortic valve

governs blood flow between the heart and the aorta, and thereby the blood vessels to the rest of

the body. The mitral and aortic valves are the ones most frequently affected by valvular heart

disease. Normally functioning valves ensure that blood flows with proper force in the proper

direction at the proper time.

In valvular heart disease, the valves become too narrow and hardened (stenotic) to open fully,

or are unable to close completely (incompetent). A stenotic valve forces blood to back up in the

adjacent heart chamber, while an incompetent valve allows blood to leak back into the chamber

it previously exited. To compensate for poor pumping action, the heart muscle enlarges and

thickens, thereby losing elasticity and efficiency. In addition, in some cases, blood pooling in the

chambers of the heart has a greater tendency to clot, increasing the risk of stroke or pulmonary

embolism. The severity of valvular heart disease varies. In mild cases, there may be no

symptoms, while in advanced cases, valvular heart disease may lead to congestive heart failure

and other complications. Valvular heart disease accounts for 10% to 20% of all cardiac surgical

procedure.

The primary causes of valve disease are age-associated calcific valve changes and inherited or

congenital conditions. The prevalence of rheumatic valve disease now is very low in the United

States and Europe because of primary prevention of rheumatic fever, although rheumatic valve

disease remains prevalent in the developing world.In addition to patients with severe valve

disease that eventually requires mechanical intervention, there is a larger group of patients with

Page 62: STUDY GUIDE THE CARDIOVASCULAR SYSTEM AND DISORDERS · study guide the cardiovascular system and disorders ... study guide the cardiovascular system and disorders ... (ekg): pemasangan

Study Guide Cardiovascular System and Disorders

Department of Medical Education - Faculty of Medicine - Universitas Udayana, 2018

62

mild to moderate disease who need an accurate diagnosis and appropriate medical

management. The most prevalent valvular heart disease is the following: 1) mitral valve

disease, 2) aortic valve disease, and 3) tricuspid and pulmonary valve disease. Most valvular

abnormalities can be managed with medical therapy, percutaneous intervention, or surgical

intervention.

Standard References:

Mann, DL et al. Braunwald’s Heart Disease, 10th ed. Philadelphia, Elsevier Saunders, 2015.

Additional reading:

Constant, Jules. Essential for Bedside Cardiology, 2nded. New Jersey, Humana Press Inc. 2003

SCENARIO:

CASE:

A 30-year-old woman complained of progressive exertional shortness of breath in the past one

year. Physical examination revealed a loud first heart sound, an opening snap and a mid-

diastolic rumbling murmur with an irregularly irregular pulse.

LEARNING TASK:

1. What diagnosis do these findings suggest?

2. What is the underlying etiology?

3. What investigation is useful and describe the findings that are consistent with the initial/

working diagnosis?

4. What pharmacological treatment does she need? Describe the mechanism of action for

the medicine(s) and why she needs it/them?

5. What other treatment does she (might) need? Please describe the indication, procedure,

and complication that might occur!

SELF DIRECTING LEARNING and SELF-ASSESSMENT

1. Describe the etiology and clinical sign and symptoms for each valvular heart disease

(aortic valve regurgitation/stenosis, mitral valve regurgitation/stenosis, tricuspid valve

regurgitation/stenosis, pulmonic valve regurgitation/stenosis)!

2. Please explain the complication of mitral stenosis!

3. Please describe the ECG findings in aortic stenosis!

4. What is the Austin-Flint murmur?

5. Please explain the indication of mitral valve replacement procedure?

Page 63: STUDY GUIDE THE CARDIOVASCULAR SYSTEM AND DISORDERS · study guide the cardiovascular system and disorders ... study guide the cardiovascular system and disorders ... (ekg): pemasangan

Study Guide Cardiovascular System and Disorders

Department of Medical Education - Faculty of Medicine - Universitas Udayana, 2018

63

LECTURE 15

ACUTE AND CHRONIC COR PULMONALE

dr. Luh Oliva Saraswati Suastika, Sp.JP, FIHA

Department of Cardiology and Vascular Medicine

AIMS:

Define the diagnosis and management of Cor Pulmonale (Pulmonary Heart Disease).

LEARNING OUTCOME:

1. Able to define the diagnosis and management of Acute Cor Pulmonale.

2. Able to define the diagnosis and management of Chronic Cor Pulmonale.

CURRICULUM CONTENT:

1. Etiology of Acute and Chronic Cor Pulmonale.

2. Pathogenesis of Pulmonary Hypertension.

3. Clinical Manifestation of Cor Pulmonale.

4. Physical Findings of Cor Pulmonale.

5. Diagnostic techniques for Cor Pulmonale.

6. Prevention and Treatment of Cor Pulmonale.

ABSTRACT:

Cor pulmonale is a common complication of pulmonary hypertension. Cor pulmonale refers to

altered structure (eg, hypertrophy or dilatation) and/or impaired function of the right ventricle that

results from pulmonary hypertension that is associated with diseases of the lung (eg, chronic

obstructive pulmonary disease), vasculature (eg, idiopathic pulmonary arterial hypertension),

upper airway (eg, obstructive sleep apnea), or chest wall (eg, kyphoscoliosis). Right-sided heart

disease due to left-sided heart disease is not considered cor pulmonale. Pulmonary

hypertension (PH) is defined as a mean pulmonary artery (PA) Pressure >20 mmHg and is

placed in the heterogeneous group of PH associated with disorders of the respiratory system

and/or hypoxemia.

Cor pulmonale tends to be chronic and slowly progressive, but it can be acute. Acute cor

pulmonale occurs when the right ventricle cannot adapt to a sudden increase in the pulmonary

arterial pressure. The increased pulmonary artery pressure may be a consequence of a new

acute process, such as pulmonary embolism, acute respiratory distress syndrome, or

progression of the chronic disease. The diagnostic evaluation of cor pulmonale is inseparable

from the evaluation of pulmonary hypertension. Cor pulmonale could be diagnosed based on

the clinical manifestation and using chest x-ray, electrocardiography, and echocardiography as

well as magnetic resonance imaging, pulmonary function testing, and right heart catheterization.

Symptoms attributable to cor pulmonale include dyspnea on exertion, fatigue, lethargy,

exertional syncope, and exertional angina. Patients with cor pulmonale have physical findings

related to both pulmonary hypertension and right-sided heart disease.

All patients with cor pulmonale should have the underlying cause of the cor pulmonale and

pulmonary hypertension treated. The treatment of cor pulmonale can be conceptualized as

having three major physiological goals: reduction of right ventricular afterload (eg, pulmonary

artery pressure), a decrease of right ventricular pressure, and improvement of right ventricular

Page 64: STUDY GUIDE THE CARDIOVASCULAR SYSTEM AND DISORDERS · study guide the cardiovascular system and disorders ... study guide the cardiovascular system and disorders ... (ekg): pemasangan

Study Guide Cardiovascular System and Disorders

Department of Medical Education - Faculty of Medicine - Universitas Udayana, 2018

64

contractility. In the cor pulmonale condition that leads to heart failure, diuretics and nitrates may

be needed to improve the condition of the patient. Oxygen supplementation is often required to

resolve the shortness of breath. Treatments of PAH have shown a dramatic change in the past

few years. Synthetic prostacyclin (epoprostenol), prostacyclin analogs, endothelin-1 receptor

antagonists and phosphodiesterase-5 inhibitors were tested in randomized controlled trials,

leading to the approval of several drugs in each class.

SELF DIRECTING LEARNING

Basic knowledge that must be known:

1. Etiology of Acute and Chronic Cor Pulmonale.

2. Pathogenesis of Pulmonary Hypertension.

3. Clinical Manifestation of Cor Pulmonale.

4. Physical Findings of Cor Pulmonale.

5. Diagnostic Techniques for Cor Pulmonale.

6. Prevention and Treatment of Cor Pulmonale.

SCENARIO: CASE 1:

A 70-year old male came to Emergency Room due to swelling on the abdomen and both legs.

The complaints were suffered since 1 month ago and become worse. He also complains of

shortness of breath and cough, that was experienced since years and usually could be resolved

by nebulizer. He used to be a heavy smoker for 30 years, with 1-2 packs cigarette per day. The

blood pressure was 120/80 mmHg; pulse rate was 110 beats-per-minute, regular. There were

wheezing at both lung field, ascites on the abdomen, and pitting edema on both legs, and

increased of jugular venous pressure. ECG revealed sinus tachycardia 110 beats-per-minute

with P pulmonal on lead II, III and aVF. The urinary production is good.

LEARNING TASK :

1. What is the most likely diagnosis?

2. Which further examinations would you recommend to support your diagnosis?

3. What is your initial treatment?

CASE 2:

A 60-year old female with obesity and cervical cancer who had undergone chemotherapy

presented to the emergency room with dyspnea on exertion, chest pain, and syncope. Initial

assessment of vital signs as follows: blood pressure 90/50, heart rate 120x/min, respiratory rate

30x/min and SaO2 86%. Her jugular venous pressure was 10 cmH2O. Laboratory work was

notable for troponin-I of 0.172 (normal <0.04 ng/mL) and NT-proBNP of 8603 (normal <125

pg/mL). Her ECG showed a S1Q3T3 pattern with large S wave in lead I and both prominent Q

waves and inverted T waves in lead III.

LEARNING TASK :

1. What is the most likely diagnosis?

2. Which further examinations would you recommend to support your diagnosis?

3. What is your initial treatment?

Page 65: STUDY GUIDE THE CARDIOVASCULAR SYSTEM AND DISORDERS · study guide the cardiovascular system and disorders ... study guide the cardiovascular system and disorders ... (ekg): pemasangan

Study Guide Cardiovascular System and Disorders

Department of Medical Education - Faculty of Medicine - Universitas Udayana, 2018

65

SELF-ASSESSMENT :

1. Please explain the risk factors of cor pulmonale!

2. What are the complications of pulmonary hypertension?

3. What is the treatment choice for acute and chronic cor pulmonale?

Page 66: STUDY GUIDE THE CARDIOVASCULAR SYSTEM AND DISORDERS · study guide the cardiovascular system and disorders ... study guide the cardiovascular system and disorders ... (ekg): pemasangan

Study Guide Cardiovascular System and Disorders

Department of Medical Education - Faculty of Medicine - Universitas Udayana, 2018

66

LECTURE 16

CONGENITAL HEART DISEASE AND ACUTE RHEUMATIC FEVER

dr. Eka Guna Wijaya, Sp.A (K)

Department of Pediatric

AIMS:

1. Describe Non-cyanotic and Cyanotic Congenital Heart Diseases.

2. Describe to diagnose and manage Acute Rheumatic Fever.

LEARNING OUTCOME:

1. Can describe, diagnose, and manage Non-cyanotic and Cyanotic Congenital Heart

Diseases.

2. Can describe, diagnose, and manage Cyanotic Congenital Heart Diseases.

3. Can describe, diagnose, and manage Acute Rheumatic Fever.

CURRICULUM CONTENS:

1. Fetal-transitional circulations.

2. To diagnose and manage Non-cyanotic Congenital Heart Diseases and its

complications.

3. To diagnose and manage Cyanotic Congenital Heart Diseases and its complications.

4. Interpret diagnostic tools for Congenital Heart Diseases.

5. The health education and prognosis of Congenital Heart Diseases.

6. Interpret diagnostic tool for Acute Rheumatic Fever.

7. Management of Acute Rheumatic Fever and its complications.

8. Prevention and rehabilitation of Acute Rheumatic Fever.

9. Health education and prognosis of Acute Rheumatic Fever.

ABSTRACT I:

Congenital Heart Disease (CHD) is a congenital malformation of the heart including a great

vessel that occurred since the baby was delivered.

A lot of kind of CHD has been recognized but ventricular septal defect, atrial septal defect,

patent ductus arteriosus were the most common finding. Tetralogy of Fallot is the commonest

one of cyanotic CHD. Obstructive lesions (pulmonary and aortic stenosis, coarctation aorta),

transposition of great artery, truncus arteriosus, Ebstein anomaly, etc were relatively rare cases.

CHD is really a dynamic disease. In mild and simple lesion such as VSD and ASD were usually

asymptomatic and half of those may undergo spontaneous closure after two years old. The

contrast in severe cases, a sign of heart failure, deep cyanosis, acidosis and another sign

express of the critical condition may exist in few hours after birth.

Severe pulmonary hypertension is a serious long-term complication of the large left-to-right

shunt. Eisenmenger syndrome may slowly develop when pulmonary artery pressure higher than

systemic pressure. The patient appeared cyanotic who previously non-cyanosis.

Left-to-right shunt hemodynamically characterized by an increase of pulmonary blood flow but

inversely decrease of systemic blood flow. Under these circumstances may lead to congestive

heart failure due to overcompensated of sympathetic and humoral stimulation.

Page 67: STUDY GUIDE THE CARDIOVASCULAR SYSTEM AND DISORDERS · study guide the cardiovascular system and disorders ... study guide the cardiovascular system and disorders ... (ekg): pemasangan

Study Guide Cardiovascular System and Disorders

Department of Medical Education - Faculty of Medicine - Universitas Udayana, 2018

67

ToF may characterize by four anatomical abnormalities: VSD, overriding of the aorta, right

ventricular hypertrophy, and pulmonary stenosis. Right-to-left shunting was seen in ventricular

level. The severity of cyanosis in ToF depends directly on the severity of pulmonary stenosis.

Growth failure is the commonest finding of significant CHD. Screening should be done in a

patient with failure of growth and development and certain syndromes to evaluate more carefully

in other to be sure is the patient having or not having CHD.

Diagnosis investigation of CHD was the following: history taking (antenatal, natal, and

postnatal), physical examination, chest radiograph, and ECG. Echocardiography is needed to

evaluate more detail of anatomical defect and cardiac function. Comprehenship management

should be performed in nursing the patient. Dental hygiene, nutritional support, psychological

aspect was a part of integrated management beyond of the medical and surgical intervention.

ABSTRACT II:

Valvular Heart Disease (VHD) is largely variated disease due to an anomaly or damaged of one

or more cardiac valves.

Anomaly most likely congenital in origin include Tricuspid Atresia, Tricuspid Steno-insufficiency

(Ebstein anomaly) mitral stenosis, mitral insufficiency, pulmonary or aortic stenosis/atresia.

The most common of VHD is Rheumatic Heart Disease and Mitral Valve Prolapse.

Diagnosis investigation like another disease: History taking, physical examination, chest X-rays,

ECG, and other specific laboratory examination. Echocardiography is a routine procedure to

evaluate more detail anatomical abnormality, severity, and cardiac function. Catheterization is

needed when valvuloplasty was indicated.

The origin and characteristic of the first and the second heart sound should be deeply

understood before identified many kinds of a pathological heart murmur.

Location, timing, quality, intensity, and transmission of heart murmur is the basic auscultatiion

modality to investigate more advance of specific valvular heart disease.

Management of VHD medically includes digitalis, diuretics, vasodilator, antithrombotic agent,

endocarditis prophylaxis, dental hygiene and nutritional support.

Rheumatic fever/ Rheumatic Heart Disease was agreed worldwide as an autoimmune disease.

Tissue hospes has a mimic antigenic structure with a certain strain of beta-hemolytic

streptococcus group a who was infected in the sharing. There is basic pathogenesis concept in

development tissue injury/ damage of susceptible host. When autoantibody was generated

insignificant number, a cross-reaction where streptococci causing agent were killed naturally by

humoral and cellular antibody but on the other hand tissue damage of the houses was also

happen because it was recognized as antigen.

Carditis and arthritis were the most frequent of a major symptom of RF/RHD.

Jones criteria were established as a definite diagnosis of rheumatic fever. Evolution was made

from the beginning in 1944 and then revised in 1956, modified in 1965, updated in 1992, finally

recommendation of WHO in 2002.

Bed rest, eradication of causing agent, inflammatory drug, and secondary prophylaxis were the

basic management of Rheumatic Fever/ Rheumatic Heart Disease

Standard References:

1. Park, MK. Pediatric Cardiology for Practitioners. 4th Ed. Philadelphia, Mosby. 2002. p

129-144, 185-189, 304-310, 311-318

Page 68: STUDY GUIDE THE CARDIOVASCULAR SYSTEM AND DISORDERS · study guide the cardiovascular system and disorders ... study guide the cardiovascular system and disorders ... (ekg): pemasangan

Study Guide Cardiovascular System and Disorders

Department of Medical Education - Faculty of Medicine - Universitas Udayana, 2018

68

SELF DIRECTING LEARNING

Basic knowledge that must be known:

1. Fetal-transitional circulations

2. Criteria diagnose and manage the Non-cyanotic Congenital Heart Diseases and its

complications

3. Criteria diagnose and manage the Cyanotic Congenital Heart Diseases and its

complications

4. The health education and prognosis of Congenital Heart Diseases

5. Interpret diagnostic tool for Acute Rheumatic Fever

6. Management of Acute Rheumatic Fever and its complications

7. Prevention and rehabilitation of Acute Rheumatic Fever

8. Health education and prognosis of Acute Rheumatic Fever.

SCENARIO: CASE 1:

Putu, 2 years old girl came to pediatric cardiology clinic with her parent with the main complain

of a persistent cough and slight dyspnea.

Physical examination:

HR: 128 x/min, RR: 44 x/min, BW: 9 kg. Positive precordial bulging, cardiac impulse was

displaced to caudolateral associated with lifting. A heart murmur has heard systolic and diastolic

phase at the upper left parasternal border.

LEARNING TASK :

1. How to know that patient having a continuous murmur?

2. What is the probable complete diagnosis clinically?

3. Is the patient should be given indomethacin?

4. What is the best diagnostic tool in this patient?

5. What kind of treatment has been recommended?

CASE 2:

Made, 9 months old baby was referred by GP to the pediatric clinic of cardiology due to

cyanosis. Physical examination looked at the baby having cyanotic at the mouth until the

tongue. Cyanotic was also seen at the fingers associated with clubbing. When auscultation just

to be done, the baby suddenly hard crying, uncontrolled for a long time and then hypercapnia

and lethargy.

LEARNING TASK:

1. What may happen to the baby?

2. What must you be done immediately to overcome this condition?

3. On auscultation, ejection systolic murmur was heard at the upper left parasternal

borderline with almost there is no heard of P2. Why?

4. What is the most probable disease may occur in the baby?

5. What do you expect from chest X-ray examination?

6. When should phlebotomy be performed base on routine blood examination?

7. Mention a lot of complication may be developed and what is the most hazard!

8. When should iron preparation be given to this patient?

Page 69: STUDY GUIDE THE CARDIOVASCULAR SYSTEM AND DISORDERS · study guide the cardiovascular system and disorders ... study guide the cardiovascular system and disorders ... (ekg): pemasangan

Study Guide Cardiovascular System and Disorders

Department of Medical Education - Faculty of Medicine - Universitas Udayana, 2018

69

CASE 3:

Komang, 10 years old boy comes with his parent to the pediatric clinic of cardiology with the

main complain of dyspnea on exertion. Coughing and palpitation were also present. Physical

examination revealed: a Malnourish boy with slight anemic. Pulse rate: 108 x/ min, RR: 24

x/min, body temperature 38 degrees C. Hyperdynamic of precordium with a displacement of

apical impulse caudolaterally with lifting positive. Holosystolic murmur was heard at cardiac

apex referred to the axilla. Diastolic murmur was also heard at the upper right parasternal

border.

LEARNING TASK

1. Base on those data, what is the most probable diagnosis?

2. What are other history and laboratory examination may be needed to support the

diagnosis?

3. Which of the cardiac valve were involved in this patient?

4. How about chest X-ray and blood pressure examination?

5. How to manage in short and long time period?

SELF ASSESSMENT

1. Please describe the hemodynamic change in PDA!

2. Please describe the pattern of blood pressure and pulse in PDA!

3. How the chest X-ray in a patient with PDA?

4. Is in large PDA you can hear diastolic flow murmur at the apex cordis? Can you explain

about that?

5. Please mention complication of PDA!

6. Please mention a few risk factor in the development of cyanotic spell!

7. Can you explain the pathomechanism of cyanotic spell?

8. Please mention the differential diagnosis of cyanotic CHD base on increase and

decrease of pulmonary blood flow!

9. Please explain what do you know about peripheral and central cyanosis!

10. Explain pathomechanism oh tissue injury in an acute rheumatic fever!

11. Mention etiology, antigenic structure and it’s cellular product!

12. Please mention mayor and minor manifestation of rheumatic fever!

13. Please mention detail pathology of rheumatic fever!

Page 70: STUDY GUIDE THE CARDIOVASCULAR SYSTEM AND DISORDERS · study guide the cardiovascular system and disorders ... study guide the cardiovascular system and disorders ... (ekg): pemasangan

Study Guide Cardiovascular System and Disorders

Department of Medical Education - Faculty of Medicine - Universitas Udayana, 2018

70

BASIC CLINICAL SKILLS

Topik BCS PIC

Pemeriksan fisik kardiovaskular 1 dr. Luh Oliva S Suastika, SpJP

Pemeriksan fisik kardiovaskular 2 dr. IB Rangga W, SpJP(K)

Elektrokardiografi dr. Putra Swi Antara, SpJP(K)

Chest imaging dr. Lisna Astuti, Sp.Rad

Resusitasi jantung paru dr. Kadek Susila SD, SpJP

IV line procedure dr. Pontisomaya Parami, SpAn, MARS

Page 71: STUDY GUIDE THE CARDIOVASCULAR SYSTEM AND DISORDERS · study guide the cardiovascular system and disorders ... study guide the cardiovascular system and disorders ... (ekg): pemasangan

Study Guide Cardiovascular System and Disorders

Department of Medical Education - Faculty of Medicine - Universitas Udayana, 2018

71

BASIC CLINICAL SKILL 1 CARDIOVASCULAR PHYSICAL EXAMINATIONS

dr. Luh Oliva Saraswati Suastika, Sp.JP, FIHA

dr. I.B. Rangga Wibhuti, Sp.JP(K), FIHA, FASE

AIMS:

Able to do a physical examination of the Cardiovascular System.

LEARNING OUTCOME:

Able to do a physical examination of the Cardiovascular System.

CURRICULUM CONTENT:

1. Evaluate general appearance

2. Blood pressure examination, the arterial pulse, the jugular venous pulse evaluation.

3. Inspection, palpation, auscultation.

ABSTRACT:

Physical examination is the procedure should be done to obtain any data from the patient. For

cardiovascular system, we should do the examination carefully. The physical examination for

the cardiovascular system consists of evaluating general appearance, blood pressure

examination, arterial pulse evaluation, the jugular venous pressure, and percussion, palpation,

auscultation, and examination for any edema.

Each of the procedure will reveal specific data from the patient. For cardiovascular system,

auscultation will play an important role in diagnosing the patient. From auscultation, we should

obtain the heart sound quality and identify any murmur present. From percussion, we should

obtain any enlargement of the heart.

SELF DIRECTING LEARNING

Basic knowledge that must be known:

1. Evaluate general appearance.

2. Blood pressure examination, the arterial pulse, the jugular venous pulse evaluation.

3. Inspection, palpation, auscultation.

Page 72: STUDY GUIDE THE CARDIOVASCULAR SYSTEM AND DISORDERS · study guide the cardiovascular system and disorders ... study guide the cardiovascular system and disorders ... (ekg): pemasangan

Study Guide Cardiovascular System and Disorders

Department of Medical Education - Faculty of Medicine - Universitas Udayana, 2018

72

CHECKLIST:

CARDIOVASCULAR PHYSICAL EXAMINATION

NO ITEMS SCORE

0 1 2

1 Evaluate General Appearance

Inspection of the skin, nails, faces, eyes, mouth

2 Blood Pressure Examination

Determine blood pressure

Rule out orthostatic hypotension, coarctation of the aorta, cardiac

tamponade

3 The Arterial Pulse

Rate and rhythm of the heart

Contour of the pulse

Amplitude of the pulse

4 The Jugular Venous Pulse

Estimate the jugular venous pressure

Evaluate the hepatojugular reflux

5 Inspection

Chest configuration

Ictus cordis

6 Palpation

Palpate the point of maximum impulse

Palpate for thrills

7 Auscultation

Auscultate the cardiac areas

The Influence of breathing

Describe any murmur present

8 Examination of Edema

Test for Edema

Score:

Page 73: STUDY GUIDE THE CARDIOVASCULAR SYSTEM AND DISORDERS · study guide the cardiovascular system and disorders ... study guide the cardiovascular system and disorders ... (ekg): pemasangan

Study Guide Cardiovascular System and Disorders

Department of Medical Education - Faculty of Medicine - Universitas Udayana, 2018

73

BASIC CLINICAL SKILL 2

ELECTROCARDIOGRAPHY

dr. I Made Putra Swi Antara, SpJP(K), FIHA

AIMS:

Able to skills for Electrocardiography (ECG), procedure of, and ECG Interpretation,

Echocardiography, Phonography, and USG Doppler.

LEARNING OUTCOME:

1. Able to do the ECG procedure.

2. Able to do ECG interpretation.

CURRICULUM CONTENS:

1. ECG procedure

2. ECG Interpretation.

ABSTRACT:

Electrocardiogram (ECG) is an examination of conduction of heart’s electrical impulse. It is

widely used and invaluable clinical tool for the detection and diagnosis of a broad range of

cardiac conditions, as well as a technique that has contributed to the understanding and

treatment of virtually every type of heart disease. Electrocardiography remains the most direct

method for assessing abnormalities of cardiac rhythm. Furthermore, the ECG is essential in the

management of major metabolic abnormalities such as hyperkalemia and certain other

electrolyte disorders, as well as in assessing drug effects and toxicities such as those caused by

digitalis, antiarrhythmic agents, and tricyclic antidepressants. It will evaluate the impulse made

by the pacemaker. The ECG will record any of important electrical activity of the heart as P

wave, QRS complex, and T wave. Each of them reflects the activity of atrium and ventricle. The

ECG changes will be associated with any electrical events occurring in the heart. The standard

clinical ECG includes recordings from 12 leads. These 12 leads include three bipolar (leads I, II,

and III), six unipolar precordial leads (leads V1 through V6), and three modified unipolar limb

leads (the augmented limb leads aVR, all, and aVF). Students should be able to analyze the

printout and interpret the result as clinical finding for diagnosis. In order to obtain a good result

of ECG, the student should be able to demonstrate skills for ECG procedure.

The procedure will consist of the preparation of the equipment and the patient, the placement of

the leads, the recording, and the interpretation. Interpretation will be done through the basic

rhythm analysis, the cardiac axis, check the limbs and chest lead for the wave morphology.

There will be some common abnormalities for ECG such as pre-excitation phenomena, bundle

branch block, hypertrophy and the most importance the myocardial ischemia. The student

should be able to analyze the cardiogram given in the course.

Echocardiography remains the most frequently used and usually the initial imaging test to

evaluate all cardiovascular diseases related to a structural, functional, or hemodynamic

abnormality of the heart or great vessels. Echocardiography uses ultrasound beams reflected by

cardiovascular structures to produce characteristic lines or shapes caused by normal or altered

cardiac anatomy in one, two, or three dimensions by M (motion)–mode, two-dimensional, or

three-dimensional echocardiography, respectively. Doppler examination and color flow imaging

Page 74: STUDY GUIDE THE CARDIOVASCULAR SYSTEM AND DISORDERS · study guide the cardiovascular system and disorders ... study guide the cardiovascular system and disorders ... (ekg): pemasangan

Study Guide Cardiovascular System and Disorders

Department of Medical Education - Faculty of Medicine - Universitas Udayana, 2018

74

provide a reliable assessment of cardiac hemodynamics and blood flow. Reliable noninvasive

hemodynamic evaluation and confident delineation of cardiovascular structures by

echocardiography have dramatically reduced the clinical necessity for hemodynamic cardiac

catheterization. Increasingly, patients undergo valvular or congenital heart surgery on the basis

of an echocardiographic diagnosis. Echocardiographic units are also being miniaturized to

become an extension of a clinician’s physical examination. In our opinion, the appreciation of

cardiac anatomy and hemodynamics by bedside echocardiography makes a physician’s clinical

evaluation, including physical examination, more relevant to the care of patients. For all

physicians who care for patients with a cardiovascular problem, it is essential to know how

echocardiographic images are obtained, what type of information echocardiography can

provide, and how it should be used for management.

Reference: Mann, DL et al. Braunwald’s Heart Disease, 10th ed. Philadelphia, Elsevier

Saunders, 2015. p. 118-132

SELF DIRECTING LEARNING

Basic knowledge that must be known:

1. ECG procedure.

2. ECG Interpretation

Preparation ECG Procedure

The group should choose one of their members to become a patient for the ECG examination.

Ask the patient to lie down on the table. In turn, each the student should perform the ECG

Examination; the student should start with patient preparation, set up the machine, recording

step, and obtaining the result.

Instruction:

1. Prepare the patient for ECG examination.

2. Set the electrocardiography appropriately.

3. Place the leads in appropriate position.

4. Start the examination.

5. Obtain the result properly.

6. Explain and give information to the patient.

7. Please, refer to the ECG Skills Checklist!

Independent Learning (ECG Interpretation)

Each group will be provided with 10 pieces of the electrocardiogram. The student should be

familiar with the analyzing step for the ECGram. It is likely that student should start by checking

the patient ID, analyze the rhythm, and identify whether there are any abnormality patterns. The

student should also be familiar with the writing technique for the ECGram’s interpretation.

Instruction

1. Analyze the ECGram given in a group. You should refer to the handout given (Analyzing

the ECG) for the interpretation!

2. Write down the interpretation made for each ECGram! Discuss the result at the wrap-up

session!

Page 75: STUDY GUIDE THE CARDIOVASCULAR SYSTEM AND DISORDERS · study guide the cardiovascular system and disorders ... study guide the cardiovascular system and disorders ... (ekg): pemasangan

Study Guide Cardiovascular System and Disorders

Department of Medical Education - Faculty of Medicine - Universitas Udayana, 2018

75

BASIC CLINICAL SKILL 3

CHEST RADIOLOGY IMAGING

dr. Lisnawati Astuti, Sp.Rad

AIMS:

Able to evaluate and result in chest x-rays.

LEARNING OUTCOME:

1. Able to evaluate chest x-rays, including evaluation of heart, lung, diaphragm, skeleton

and soft tissues.

2. Able to result in chest x-rays.

CURRICULUM CONTENT:

1. Able to evaluate chest x-rays, including evaluation of heart, lung, diaphragm, skeleton

and soft tissues.

2. Able to result in chest x-rays.

ABSTRACT:

Chest imaging is an important evaluation that supports the diagnosis procedure. The student

should be able to evaluate chest x-rays, including evaluation of heart, lung, diaphragm, skeleton

and soft tissues. After evaluation, the student should be able to write down the result in a given

format. Some emergency case, need rapid chest x-rays evaluation. By this training, we hope

that the student will be able to do such important skill. There are steps in evaluating the chest x-

rays, it is systematic steps. The student should be mastered. For the cardiovascular system, the

chest imaging will be posterolateral, lateral, oblique projection. The student should evaluate the

heart size; identify any enlargement, the condition of the lung – any edema, arterial and venous

hypertension.

The imaging investigation of the heart may be considered under the following:

1. Chest X-ray

2. Computed tomography (CT-scan)

3. Magnetic resonance imaging (MRI)

4. Echocardiography

5. Angiocardiography

6. Cardiac catheterization

7. Isotope scanning

Chest X-ray remains the valuable cardiac investigation in clinical practice.

The radiologic method used in the roentgen cardiac examination:

1. Posteroanterior projection, PA/AP

2. Lateral Projection

3. Right anterior oblique projection (RAO)

4. Left anterior oblique projection (LAO)

Increase in cardiac size is the most consistent indication of cardiac disease.

Page 76: STUDY GUIDE THE CARDIOVASCULAR SYSTEM AND DISORDERS · study guide the cardiovascular system and disorders ... study guide the cardiovascular system and disorders ... (ekg): pemasangan

Study Guide Cardiovascular System and Disorders

Department of Medical Education - Faculty of Medicine - Universitas Udayana, 2018

76

SELF DIRECTING LEARNING

Basic knowledge that must be known:

Able to evaluate chest x-rays, including evaluation of heart, lung, diaphragm, skeleton and soft

tissues.

LEARNING TASK I:

Chest imaging: Cardiovascular System

Preparation

There would be 10 set of the light cast with a single x-ray film. The group should discuss the x-

ray film and write down the result in a piece of A4 paper. You should notice the time limit for

each film. It would be at least 5 – 10 minutes of discussion for each film. The group should

move to another x-rays film after complete the discussion and write down the result.

Instruction

1. The group should read the case available before evaluate the x-ray photo. What is the

main complaint of the patient?

2. The group should evaluate the x-rays photo systematically!

3. Write down the group result on a piece of A4 paper.

4. Move to another x-ray and you should repeat the step 1 till 3 (each group should read all

photos available).

Standard References:

Roentgen Signs in Diagnostic Imaging Isadore Meschan

Page 77: STUDY GUIDE THE CARDIOVASCULAR SYSTEM AND DISORDERS · study guide the cardiovascular system and disorders ... study guide the cardiovascular system and disorders ... (ekg): pemasangan

Study Guide Cardiovascular System and Disorders

Department of Medical Education - Faculty of Medicine - Universitas Udayana, 2018

77

BASIC CLINICAL SKILL 4

dr. Pontisomaya Parami, SpAn, MARS

AIMS:

1. Able to practice skill routine clinical procedure: Intravenous line (IV line).

2. Able to practice skill routine clinical procedure: venipuncture.

LEARNING OUTCOME:

Able to two skills should be trained: IV cannulation and venipuncture.

CURRICULUM CONTENS:

1. Technique of venipuncture.

2. Aseptic procedure.

ABSTRACT :

The doctor should be able to draw blood in a field setting as a part of disease investigation and

therapy. Appropriate equipment and supplies should including the following gloves, aseptic kit,

bandage, tourniquet, vacutainer tubes, or spuit and the container. The complete technique of

venipuncture is contained in the Venipuncture Evaluation Checklist. It will cover the skills in

preparation of the doctor and the patient, aseptic procedure, the preparation of the kit,

communicate the procedure to the patient, the patient preparation, the insertion technique of the

needle, the blood collection, evaluation for any bleeding, and cleaning the work area after the

procedure.

The checklist will vary from one to another, you should use the checklist as aide-memoir (or

reminder) of the element skills to be done. Basically, there would be four main steps in doing the

venipuncture and IV line cannulation. It would explain the procedure, prepare the equipment

and position the patient, select an appropriate site, use standard precaution, and reach the goal

(obtain the adequate specimen and a good technique for cannulation).

SELF DIRECTING LEARNING

Basic knowledge that must be known:

1. The complete technique of venipuncture

2. Aseptic procedure

3. The preparation of the kit

4. Communicate the procedure to the patient

5. The patient preparation

6. The insertion technique of the needle, the blood collection

7. Evaluation of any bleeding

8. Cleaning the work area after the procedure.

Page 78: STUDY GUIDE THE CARDIOVASCULAR SYSTEM AND DISORDERS · study guide the cardiovascular system and disorders ... study guide the cardiovascular system and disorders ... (ekg): pemasangan

Study Guide Cardiovascular System and Disorders

Department of Medical Education - Faculty of Medicine - Universitas Udayana, 2018

78

Training Task

Venipuncture and IV Line Procedure

Preparation

We will provide the student with Multipurpose Injection Arm and IV line mannequin needed for

IV line procedure training. It would be at least one mannequin for every two groups.

Groups should prepare one infusion set, ringer lactate infusion fluid, antiseptic set, and tape.

Each student should bring their own IV needle (G-21), a syringe (3 cc), and bring a glove for the

antiseptic procedure.

Instruction

1. There would be two skills should be trained in this session, IV cannulation and

venipuncture.

2. You have to prepare the set for the procedures, prepare the mannequin, needle, the

infusion set, and the infusion fluid.

3. Demonstrate how will you explain the procedure to the patient, the technique and the

complication might happen.

4. Demonstrate the technique for IV cannulation and venipuncture. Please notice the

position of your finger, the angle, and how to evaluate whether the needle inserted

properly.

5. Refer to the checklists (Venipuncture and IV Cannulation) for any details!

6. Ask any comment and score for your performance from your groups based on the

checklist!

Page 79: STUDY GUIDE THE CARDIOVASCULAR SYSTEM AND DISORDERS · study guide the cardiovascular system and disorders ... study guide the cardiovascular system and disorders ... (ekg): pemasangan

Study Guide Cardiovascular System and Disorders

Department of Medical Education - Faculty of Medicine - Universitas Udayana, 2018

79

EVALUATION FORM OF THE CARDIOVASCULAR SYSTEM AND DISORDERS

Please fill the form according to the real condition. This evaluation will not influence your final

block result.

Please cross on the scoring column that suitable with your judgment.

No. Point being evaluated Score

1 2 3 4 5

A Topic

1. Approach to patient with cardiovascular disease

2 Cardiovascular Pharmacology

3 The formation of anomalies of the heart and great

vessels.

4 Cardiac markers

5 Cardiac rehabilitation

6 Acute Coronary Syndrome

7 Stable Angina Pectoris

8 Acute heart failure

9 Chronic Heart Failure

10 Hypertension

11 Hypertensive urgency & emergency

12 Arrhythmias

13 Valvular Heart Disease

14 Cor Pulmonale

15 Non-cyanotic & Cyanotic CHD & Acute Rheumatic

Fever

16 Common peripheral vascular disease

17 CV Physical Examination

18 ECG

19 Chest Imaging

20 CV Physical Examination 2

21 Cardiopulmonary resuscitation

22 IV line Procedure

B Learning strategy

1 Lecture

2 Independent learning

3 Small group discussion

4 Practical

5 Case-based learning

6 Problem-based learning

7 Learning task

8 Self-assessment

Page 80: STUDY GUIDE THE CARDIOVASCULAR SYSTEM AND DISORDERS · study guide the cardiovascular system and disorders ... study guide the cardiovascular system and disorders ... (ekg): pemasangan

Study Guide Cardiovascular System and Disorders

Department of Medical Education - Faculty of Medicine - Universitas Udayana, 2018

80

C Lecturer

1. Prof. Dr. dr. I Wayan Wita, Sp.JP(K), FIHA,

FAsCC

2. dr. Ni Putu Ekawati, Sp.PA, M.Repro

3. Dr. dr. Wiradewi, Sp.PK

4. dr. Agung Nova Mahendra, M.Sc

5. dr. I Made Junior Rina Artha, Sp.JP(K), FIHA,

FAsCC

6. dr. I.B. Rangga Wibhuti, M.Biomed, Sp.JP(K),

FIHA, FASE

7. dr. I Made Putra Swi Antara, Sp.JP(K), FIHA

8 dr. I Kadek Susila Surya Darma, M. Biomed,

Sp.JP, FIHA

9 dr. A.A. Ayu Dwi Adelia Yasmin, M. Biomed,

Sp.JP, FIHA

10. dr. Nyoman Wiryawan, SpJP(K), FIHA

11. dr. Luh Oliva Saraswati Suastika, Sp.JP, FIHA

12. dr. Hendy Wirawan, SpJP

13. dr. Eka Guna Wijaya, Sp.A(K)

14. dr. Pontisomaya Parami, Sp.An. MARS

15. dr. Luh Kamiati, Sp.KFR

16. Dr. dr. Semadi, Sp.B, Sp.BTKV

D Facilitator

1 Name of your group facilitator:

E Assessment

1 Time provides

2 Suitability of question with topic given

Score:

1. Bad or not suitable for expectation

2. Insufficient or inadequate with expectation

3. Sufficient or inadequate with expectation

4. Good or suitable with expectation

5. Excellent or exceed expectation

Page 81: STUDY GUIDE THE CARDIOVASCULAR SYSTEM AND DISORDERS · study guide the cardiovascular system and disorders ... study guide the cardiovascular system and disorders ... (ekg): pemasangan

Study Guide Cardiovascular System and Disorders

Department of Medical Education - Faculty of Medicine - Universitas Udayana, 2018

81

The problem you found during Block Cardiovascular System and Disorders for each point

evaluated above:

Topic

Learning strategy

Lecturer

Facilitator

Assessment

Your suggestion/ input:

Topic

Learning strategy

Lecturer

Facilitator

Assessment

Page 82: STUDY GUIDE THE CARDIOVASCULAR SYSTEM AND DISORDERS · study guide the cardiovascular system and disorders ... study guide the cardiovascular system and disorders ... (ekg): pemasangan

Study Guide Cardiovascular System and Disorders

Department of Medical Education - Faculty of Medicine - Universitas Udayana, 2018

82

REFERENCES

Student Standard References :

1. Moore KL, Agur AMR: Essential Clinical Anatomy, 3rd ed. Philadelphia, Lippincott & Wilkins,

2007.

2. Sadler TW: Langman’s Medical Embryology, 10th ed. Philadelphia, Lippincott & Wilkins,

2006.

3. Gartner LP, Hiatte JL: Color Textbook of Histology, 2nd ed. Philadelphia, WB Saunders

Company, 2001.

4. Guyton AC: Textbook of Physiology, 11st ed. Philadelphia, WB.Saunders Company, 2006

5. Fox S.I.: Human Physiology, 9th ed. New York, McGraw-Hill, 2006

6. Kumar V, Cotran RS, Robbins SL: Robbin’s Basic Pathology, 7th ed. Philadelphia, Saunders,

2003

7. Trevor AJ, Katzung BG, Masters: Katzung & Trevor’s Pharmacology, 7th ed. New York,

Lange Medical Book’s/Mc.Graw-Hill, 2005.

8. Park MK. Pediatric Cardiology for Practitioners. 4th Ed. Philadelphia, Mosby. 2002.

9. McPhee, S.J., Papadakis, M.A., Current Medical Diagnosis & Treatment. 47th ed. New York,

Lange Medical Book`s/The McGraw-Hill Companies, 2008.

Additional Student References :

1. A2: Moore KL, Dalley AF: Clinically Oriented Anatomy, 4th ed. Philadelphia Lippincott &

Wilkins, 1999.

2. H2: Fowcett DW, Jensh RP: Bloom & Fawcett’s Concise Histology, 2nd ed. London,

Arnold. 2002.